Naked Science Forum

Non Life Sciences => Physics, Astronomy & Cosmology => Topic started by: jeffreyH on 21/04/2018 20:03:29

Title: Can we measure the one way speed of light?
Post by: jeffreyH on 21/04/2018 20:03:29
The setup: Two long sheets of photographic paper on two separate conveyor systems. These move at the same rate, one clockwise and the other counter clockwise. Photons are fired at them from opposite directions. One set of photons each at each film. The spacing between photons is identical in each case. The marks on the paper should differ in spacing if the one way speed of light differs. Of course the whole apparatus must be long enough and moving fast enough for there to be a detectable effect. If this shows no difference then the one way speed of light idea is dead and buried.
Title: Re: Can we measure the one way speed of light?
Post by: wolfekeeper on 22/04/2018 00:12:16
This is dead and buried. The one way speed of light is always 'c'.

You can explain this result in various ways. The way I prefer is to point out that if Newtonian mechanics is correct, and Maxwell's equations are correct as well, then given that matter is held together with electromagnetic forces, with extreme care you can derive special relativity.

The way it works is that you can show that clocks must run slow when they start to move, and rulers get Lorentz contracted, and further, due to the time dilation that clocks have there's inevitably a systematic difference in time along any length in the direction of travel ("lack of simulteneity").

If you put those three effects together you can show that the one-way speed of light is completely independent of speed, as measured from any frame of reference, moving or not.
Title: Re: Can we measure the one way speed of light?
Post by: kpvats on 01/05/2018 16:29:17
If you mean practical measurement of one way speed of light, then it is going to require synchronized clocks, otherwise, it is not possible.
Title: Re: Can we measure the one way speed of light?
Post by: PmbPhy on 02/05/2018 09:51:18
Hi Jeff- Great question as always. Not only can it be done but it has been done. Its required knowledge for very long baseline interferometry (VLBI) in radio astronomy. It requires having two synchronized clocks at each location of each station, Dave Cooper thought it was impossible but to me it merely seems that he lacks a full understanding of clock synchronization, There are more than one way to synchronize clocks, People always seem to think that it its not done the way Einstein explained i then its not a real synchronization. On the contrary. Two atomic clocks can be synchronized at one place and then each clock is transported at a sloe speed (driving a car perhaps) and the clocks will still read the same time within experimental limits. If the one way speed of light was different depending on direction then inconsistencies with be found in the data, None were found,
Title: Re: Can we measure the one way speed of light?
Post by: jeffreyH on 02/05/2018 11:58:55
Thanks Pete. Do you have any references to papers on the subject?
Title: Re: Can we measure the one way speed of light?
Post by: PmbPhy on 02/05/2018 13:59:11
Thanks Pete. Do you have any references to papers on the subject?
No. But I an scan that portion of Ohanian's text and put them in a PDF file and upload it here. Isn't the 21st century marvelous!! :D
Title: Re: Can we measure the one way speed of light?
Post by: David Cooper on 20/05/2018 21:51:25
The setup: Two long sheets of photographic paper on two separate conveyor systems. These move at the same rate, one clockwise and the other counter clockwise. Photons are fired at them from opposite directions. One set of photons each at each film. The spacing between photons is identical in each case. The marks on the paper should differ in spacing if the one way speed of light differs. Of course the whole apparatus must be long enough and moving fast enough for there to be a detectable effect. If this shows no difference then the one way speed of light idea is dead and buried.

I can't work out from that what your alignments are for the paths of the photons relative to these conveyor systems. Should I assume that its a shallow angle from the end aimed with high precision half way along sheet of photographic paper? I'm also imagining that the whole apparatus is moving along through space in the same direction as the conveyor systems are aligned, so one sheet of photographic paper is moving forwards more quickly than the other.

If that's what you had in mind, then you have to length-contract your photographic paper appropriately for its speed of travel through space, and once you've applied the right amount of contraction, you'll find that the light pulses hit the paper in such a way that when you stop the experiment and measure the distances between them carefully, they'll have identical spacings between them on both sheets of paper (i.e. the same separation distance on both sheets). [If you're shooting both sheets with photons from both ends, you'll have two lots of dots recorded on each sheet with different separations, but these two separation values will be the same for the patterns of dots on both sheets of paper.] You can't cheat relativity. Incidentally, both conveyor belts will length-contract equally, but the contraction applying to the paper on different sides will not match - there will be more paper on one side than the other at any moment in time while the system is moving through space.
Title: Re: Can we measure the one way speed of light?
Post by: David Cooper on 20/05/2018 22:15:18
Dave Cooper thought it was impossible but to me it merely seems that he lacks a full understanding of clock synchronization,

What did I think was impossible? If you're going to name someone and attribute a failing to them, spell out clearly what that error is and link to some evidence to back your claim. There is no difficulty in synchronising clocks (using a wide variety of methods), except for one little problem: they are only genuinely synchronised if you're synchronising them for the absolute frame (which you can't identify).
Title: Re: Can we measure the one way speed of light?
Post by: yor_on on 26/05/2018 21:38:26
Well David, Don't know a 'absolute frame', but if you by it might mean being in a 'absolute' same frame of reference then I think even a slow moving car will fail that requirement, unless both are in it at a 'same frame of reference' at all times. Then again, the question reminds me a lot of the idea of what I think is called 'weak experiments', where you expect something intermediate to have no discernible effect on the main process/system/interaction (whatever that might imply) you want to observe.
Title: Re: Can we measure the one way speed of light?
Post by: Bill S on 26/05/2018 21:55:57
Yor_on; I'm confused (happens often).  Are you saying that the experiment to which Pete refers would be classed as a "weak experiment"?
Title: Re: Can we measure the one way speed of light?
Post by: yor_on on 26/05/2018 22:16:05
Yes, that's my view of it. Doesn't mean it's 'wrong'. It's about you defining your 'system', and from there draw conclusions, as light actually 'moving', not only 'propagating'.
Title: Re: Can we measure the one way speed of light?
Post by: David Cooper on 27/05/2018 19:53:27
Well David, Don't know a 'absolute frame', but if you by it might mean being in a 'absolute' same frame of reference then I think even a slow moving car will fail that requirement, unless both are in it at a 'same frame of reference' at all times. Then again, the question reminds me a lot of the idea of what I think is called 'weak experiments', where you expect something intermediate to have no discernible effect on the main process/system/interaction (whatever that might imply) you want to observe.

"Absolute frame of reference" has a very specific meaning, being the frame in which light travels at c in all directions relative to a stationary object. Clock ticks are only genuinely simultaneous if the clocks synchronised for that frame, but because we can't identify that frame, we just have to pick a convenient frame and synchronise them for that instead. I took part in a long discussion about this in another thread somewhere (which I can't now find), and I assume I'm being accused of making some kind of error there, but what I said there stands, so the error is an imaginary one either based on someone's misunderstanding of what was said there or on a mangled recollection. This accusation of an error comes from someone who ended his post by saying:-

Quote
If the one way speed of light was different depending on direction then inconsistencies with be found in the data, None were found,

That shows a complete failure to understand what LET and the MMX say about how the maths of this works, because no inconsistencies should be found in the data. This has been understood for a very long time, but still some people have not caught up with the facts.
Title: Re: Can we measure the one way speed of light?
Post by: jeffreyH on 27/05/2018 21:41:13
David makes an excellent point above about the absolute frame of reference. It is subtle but necessary to understand. I will have more to say when I have some free time.
Title: Re: Can we measure the one way speed of light?
Post by: Bill S on 27/05/2018 22:25:57
I’m obviously going to have to look again at the idea of an absolute frame of reference. 

My understanding is that an absolute frame of reference is some fixed reference frame that every observer in the universe would agree is at rest, and that relativity ruled this out.

Quote
"Absolute frame of reference" has a very specific meaning, being the frame in which light travels at c in all directions relative to a stationary object.

I thought that light travelled at c (in a vacuum) relative to any stationary (or relatively moving) object.  Wouldn’t that mean that the whole Universe was an absolute frame of reference?
Title: Re: Can we measure the one way speed of light?
Post by: jeffreyH on 27/05/2018 22:47:13
Don't let what I say sway you. Challenge me!
Title: Re: Can we measure the one way speed of light?
Post by: alancalverd on 27/05/2018 23:01:12
If VAB ≠ VBA then the value measured by any experiment will depend on the direction of AB.

This is not found by experiment.

Therefore the "one-way" speed of light  is the same as the "two-way" speed.
Title: Re: Can we measure the one way speed of light?
Post by: jeffreyH on 28/05/2018 10:08:52
In an inertial frame, and without looking for external indicators, we cannot tell if we are moving or in what direction that motion may be. Galaxies that recede from us will shift their light into the red end of the spectrum. They do tell us which way they are traveling. Would an object moving away inertially also red shift its light or is acceleration a necessity?
Title: Re: Can we measure the one way speed of light?
Post by: PmbPhy on 28/05/2018 13:12:48
Jeff - Sorry for the delay but here's what I promised:
http://www.newenglandphysics.org/other/Ohanian.pdf

Suggestion: Don't confused something being tested with something being proved. Physics is not about proving things: See: http://www.newenglandphysics.org/common_misconceptions/Alan_Guth_04.mp4
Title: Re: Can we measure the one way speed of light?
Post by: David Cooper on 28/05/2018 18:45:01
My understanding is that an absolute frame of reference is some fixed reference frame that every observer in the universe would agree is at rest, and that relativity ruled this out.

There is no such universal absolute frame - the expansion of the space fabric means that the absolute frame of reference at one location isn't necessarily the same frame as the absolute frame of reference at another location far away. The absolute frame is the frame at any point where a stationary object is stationary relative to the local fabric and where light passes that object in all directions at c relative to it.

Quote
I thought that light travelled at c (in a vacuum) relative to any stationary (or relatively moving) object.  Wouldn’t that mean that the whole Universe was an absolute frame of reference?

Light travels at c (in a vacuum and outside of any gravity well) relative to any stationary object, but two stationary objects a long way apart can be moving relative to each other due to the expansion of space, so the whole universe cannot have a single absolute frame of reference other than within an exterior space within which our space fabric is expanding, but it's beyond the ability of physics to access that at the moment, so it moves into mere speculation. Many people will tell you that discussing an absolute frame is also a step into mere speculation, but a correct interpretation of the Michelson-Gale-Pearson experiment says otherwise.
Title: Re: Can we measure the one way speed of light?
Post by: David Cooper on 28/05/2018 18:51:54
If VAB ≠ VBA then the value measured by any experiment will depend on the direction of AB.

This is not found by experiment.

Therefore the "one-way" speed of light  is the same as the "two-way" speed.

Wow - you're still clinging to that old mistake after all this time! Have you still not bothered to crunch the numbers in the way LET tells you you should so that you take into account length contraction (caused by the phenomenon of relativistic mass) and clocks being slowed by their movement through space (due to the lengthened round-trip communication distances)? When you do that, you find that no experiment will be capable of detecting any difference no matter how widely the one-way speed of light varies relative to the apparatus in opposite directions.
Title: Re: Can we measure the one way speed of light?
Post by: Bill S on 28/05/2018 20:37:03
Quote
There is no such universal absolute frame
Agreed.
Quote
The absolute frame is the frame at any point where a stationary object is stationary relative to the local fabric….
So, an “absolute frame” is a local feature?  Doesn’t that seem an odd use of the term “absolute”?
 
Quote
….and where light passes that object in all directions at c relative to it.
Isn’t that; everywhere “in a vacuum and outside of any gravity well”?
Quote
so the whole universe cannot have a single absolute frame of reference…

OK with that.
Quote
….other than within an exterior space within which our space fabric is expanding
You’ve lost me.
Title: Re: Can we measure the one way speed of light?
Post by: David Cooper on 28/05/2018 22:45:28
So, an “absolute frame” is a local feature?  Doesn’t that seem an odd use of the term “absolute”?

The name "absolute frame" is historical. The word "atom" also means indivisible, but we don't trip over that as we don't base the modern definition on the name.

Quote
Isn’t that; everywhere “in a vacuum and outside of any gravity well”?

Yes.

Quote
Quote
….other than within an exterior space within which our space fabric is expanding
You’ve lost me.

There could be an absolute frame of reference in an external space which contains our space fabric where the same frame of reference applies at every location in our space fabric, but there is no such frame within our space fabric. The simplest way to understand how this is mathematically possible is to imagine our space fabric as being the surface of an expanding 3D bubble which exists within a 4D exterior space. The speed of light through the surface of the 3D bubble is the same on every part of the bubble, and it's also the same relative to the 4D exterior space. Using the 4D space, there is a universal absolute frame which governs the speed of light through the entire 3D space fabric contained in the 4D space, but the frames of reference in the 3D space are all curved, so the absolute 3D curved frame at one point in the bubble is not the same frame as the absolute 3D curved frame at any other point.
Title: Re: Can we measure the one way speed of light?
Post by: alancalverd on 28/05/2018 22:56:31
Wow - you're still clinging to that old mistake after all this time! Have you still not bothered to crunch the numbers in the way LET tells you you should so that you take into account length contraction (caused by the phenomenon of relativistic mass) and clocks being slowed by their movement through space (due to the lengthened round-trip communication distances)? When you do that, you find that no experiment will be capable of detecting any difference no matter how widely the one-way speed of light varies relative to the apparatus in opposite directions.
The classic undergraduate experiment only uses one clock, which has no idea where the light is coming from or going to.

Maxwell's equations demonstrate c to be constant from first principles of electrodynamics, which don't involve light or motion at all.
Title: Re: Can we measure the one way speed of light?
Post by: Bill S on 29/05/2018 00:11:01
Quote from: Jeffrey
Don't let what I say sway you. Challenge me!

I can't promise not to let what you say "sway" me.  I respect your views.  However, I'll not be slow to challenge anything, as long as I understand enough to launch a  reasonable challenge.  :)
Title: Re: Can we measure the one way speed of light?
Post by: alancalverd on 29/05/2018 08:57:14
AFAIK the entire concept of length contraction, time dilation etc arises from the premise that c is independent of direction, so you can't use relativity to suggest that c isn't independent of direction, and any experiment that confirms a relativistic prediction implies that c is indeed constant.
Title: Re: Can we measure the one way speed of light?
Post by: David Cooper on 29/05/2018 20:10:53
The classic undergraduate experiment only uses one clock, which has no idea where the light is coming from or going to.

Which experiment?

Quote
Maxwell's equations demonstrate c to be constant from first principles of electrodynamics, which don't involve light or motion at all.

We've covered this before - your measurements of the distances that have to be fed into the equations have to be made in relation to a specific frame of reference, and the answers that you get out of it then assert that the frame you chose has light moving at c relative to it in all directions, but all that's happening is that you're getting the same frame back that you fed in through your distance measurements. The real distances vary depending on whether you're moving or not - we aren't dealing with a static system here, but forces which act at speed c across a distance, and that distance varies depending on how far the force carriers actually have to travel through space to complete the trip. A naive understanding (i.e. a misunderstanding) of what you're doing when you measure these distances leads to you generating incorrect conclusions, and having made this error here, you extend it to every other case and blind yourself to the reality that you cannot measure the speed of light relative to any apparatus in a single direction, and that the actual relative speeds can vary.

Quote
AFAIK the entire concept of length contraction, time dilation etc arises from the premise that c is independent of direction, so you can't use relativity to suggest that c isn't independent of direction, and any experiment that confirms a relativistic prediction implies that c is indeed constant.

The concept of length contraction came out of the result of the MMX and it led to Lorentz's relativity (LET). The "time dilation" again comes out of measurements based on the application of LET, so in both cases these come from relativity and are based on c being able to vary relative to the apparatus. All the experiments produce results fully compatible with LET.
Title: Re: Can we measure the one way speed of light?
Post by: yor_on on 31/05/2018 14:49:03
What one can say David is taking the idea of a spatial measurement (e.g a distance) as something unchangingly given may not be the best of ideas. According to relativity a  'rod' one find a length too, will be a result of ones frame of reference relative whatever frame the rod is in. The same goes for 'relative speeds'. In Relativity you have two kinds, uniform motion and accelerations. Both will have a impact on ones definition of the length of that rod.

Then you have 'being at rest' which creates a system in where, let's use the MMX https://en.wikipedia.org/wiki/Michelson%E2%80%93Morley_experiment , in where the mirrors are at rest with each other.

what sort of conclusions one can draw from that one is that no matter what 'relative speed' we might want to give Earth 'moving' with the solar system through the universe, the turning table they used showed no difference of 'speed of light' for that experiment. And that's the most interesting thing as it gives us a 'constant' called 'c'.

it's a constant
=

The whole point of something 'being at rest', as this 'system of mirrors' the MMX used, is that you now can disregard any question of 'distance'. It doesn't matter as long as they don't move relative each other. The 'system' is in a same frame of reference. But the Earth on which it was mounted has 'relative speeds', and depending on what you choose to measure it against, the sun, the galactic 'center', whatever, you then find different 'relative motions'. If that mattered the speed of light wouldn't be a constant.
Title: Re: Can we measure the one way speed of light?
Post by: guest4091 on 31/05/2018 17:17:51
This is dead and buried. The one way speed of light is always 'c'.
Yet it keeps appearing, like sightings of Elvis!
Title: Re: Can we measure the one way speed of light?
Post by: guest4091 on 31/05/2018 17:24:47
AFAIK the entire concept of length contraction, time dilation etc arises from the premise that c is independent of direction, so you can't use relativity to suggest that c isn't independent of direction, and any experiment that confirms a relativistic prediction implies that c is indeed constant.
May I add,independent of direction, implies independent of the source.
Title: Re: Can we measure the one way speed of light?
Post by: David Cooper on 31/05/2018 18:28:09
If that mattered the speed of light wouldn't be a constant.

It isn't a constant - the constant is merely the measured speed of light and not the actual speed of light.
Title: Re: Can we measure the one way speed of light?
Post by: David Cooper on 31/05/2018 18:46:45
This is dead and buried. The one way speed of light is always 'c'.

Yet it keeps appearing, like sightings of Elvis!

It keeps appearing because people keep making unsound assertions about the one way speed of light always being c, and they're basing that on experiments that fail to prove what's being claimed of them. No experiment has ever shown anything about the the speed of light in a single direction relative to the apparatus other than that it's somewhere between 2c and -2c.

What is it with people that they ignore a proof that goes against their beliefs? Which part of my analysis of the MGP experiment don't they understand? We have a ring of fibre optic cable being rotated round the centre of the ring such that the light doing a complete circuit of the loop travels faster relative to the material of the cable in one direction than the other. We can make it red light going clockwise through the ring and blue light going anticlockwise while the ring rotates anticlockwise, so the red light is moving faster relative to that material of the ring than the blue light. It doesn't matter if the ring is stationary on average or moving through space at any speed you care to name; on average, the red light is moving faster through each part of the ring than the blue light relative to the material there. Any attempt to make them equal conflicts with the result of MGP.

How do Einsteinists defend against this? They make up bogus rules about rotating frames of reference which are incompatible with the rules of non-rotating frames, and then they never bother to check to see if they're compatible or not. They aren't compatible - I showed that by mapping the inertial circuit to a non-inertial circuit and explained why the rules of non-rotating frames have to apply to rotating frames too without producing contradictions: the two frames would have to be compatible for the rotating frame rules to be valid. They aren't compatible though, so the rules for the rotating frame are not valid. The speed of light relative to an object has thus been proved to be different in different directions in such cases (and by extension, in most cases), and SR has been mathematically disproved. None of you have been able to take this on to try to show where my proof fails. If you've drawn a blank, why not call in the mathematicians and then we can bury SR officially and rescue physics from its biggest mistake.
Title: Re: Can we measure the one way speed of light?
Post by: guest4091 on 31/05/2018 19:43:26
The concept of length contraction came out of the result of the MMX and it led to Lorentz's relativity (LET). The "time dilation" again comes out of measurements based on the application of LET, so in both cases these come from relativity and are based on c being able to vary relative to the apparatus. All the experiments produce results fully compatible with LET.
The concept of lc was developed by Lorentz and Heaviside about 1888. Fitzgerald suggested the idea as a solution for the MMx. The coordinate transformations CT, were deveoped by Lorentz based on an unobservable ether. The identical CT were developed by Einstein without an ether.
They both produce the same results, with light speed being measured as c.
Title: Re: Can we measure the one way speed of light?
Post by: guest4091 on 31/05/2018 19:49:16
Can we measure the one way speed of light
part 1

Let's examine the simplest argument first.
Since matter in motion is restricted to <c, a single observer cannot be present at both the emission event and detection event of a light signal, for a 1-way path. 

We can consider the 1905 paper, par. 1, and substitute (our events) where Einstein considers an A-time for A (local emission), and a B-time for B (remote detection), but no common time. He then introduces his synchronization convention. It's critical to any argument concerning c, that the 2-way path, using synchronized clocks (c1 and c2) is symmetrical by definition. Observer A, who perceives a pseudo rest frame for himself (right drawing), would expect the outbound time to equal the inbound time, or equal path lengths. As the graphic shows (left drawing), the axis of simultaneity (s1-s2) is an abstract mathematical device. The A frame is actually moving parallel to the U reference frame. This is another instance of abstraction altering the identity of something. Time becomes a line, and all lines are equal, and to some, the lines become reality .

Notice the 2-way light path for +x (0,s2,D) is the same length as for -x, but with the long and short parts in reverse order. If light speed varied for the long part, the time would vary equally for both + and - directions, and thus be undetectable.
 [ Invalid Attachment ]
Title: Re: Can we measure the one way speed of light?
Post by: guest4091 on 31/05/2018 20:00:47
can we measure the one way speed of light
part 2
And then there's Newton.
Newton's first rule (no one knows the law) of motion: "A body remains at rest, or if in motion, remains in motion at a constant speed in a straight line, unless acted upon by an external force."
Here Newton, in typical two valued logic, defines 'rest' and 'motion' as opposites/complements, eg., light-dark, hot-cold, sweet-sour, etc. The earth and its population are moving relative to the sun at approx. 30 km/sec, yet we never consider this in our daily activities. The point is, we are at rest relative to the earth, yet simultaneously in motion relative to the sun. It seems a contradiction of terms.
If Newton's first rule is modified to: "A body is at rest relative to second body, if both have the same velocity", then all bodies are in motion, and rest is just a special case of motion. It works well in a dynamic universe.
The purpose of this idea is to avoid the need to search for the elusive 'absolute rest frame'.
Motion is quantifiable via the definitions for velocity, "the rate of change of position and direction, per unit of time". Rest is not quantifiable. Object 1 can have greater speed than object 2, but cannot have more or less 'rest' than object 2. Complementary properties are measured indirectly. From the examples above, the value of darkness can be expressed as a standard d, minus the value of detected light, dryness depends on the measurable value of wetness. etc. This is a form of "fuzzy logic", a scale of gradations vs an 'all or nothing'/ 'black or white' logic.
Science can only study things it can measure, which excludes 'rest'.
Title: Re: Can we measure the one way speed of light?
Post by: yor_on on 01/06/2018 03:36:02
David, you have to differ between the actual experiment and the abstractions you might infer from it.  " It isn't a constant - the constant is merely the measured speed of light and not the actual speed of light. "


If you have a experiment testing the speed of light, and there are several, and you find it (and every ones else experiments) to give you a same result, then that is a fact.
Title: Re: Can we measure the one way speed of light?
Post by: yor_on on 01/06/2018 03:50:41
The rest of it is actually open for interpretations still, as long as you can fit it to the experiments. Myself I like the idea of light not 'propagating' at all, instead being emanations in a 'field'. But that doesn't mean I know how to test that idea, and prove it :) I just think it might fit. If you really expect yourself to have a experiment proving different speeds of light you should construct it and then present your results.
Title: Re: Can we measure the one way speed of light?
Post by: David Cooper on 01/06/2018 20:10:36
David, you have to differ between the actual experiment and the abstractions you might infer from it.

I agree - if you do the experiments and then build conclusions based on the results where you apply bad reasoning and state that you've measured the one-way speed of light and that the figure you've produced actually is the speed of light relative to the apparatus, you have departed from science.

Quote
If you have a experiment testing the speed of light, and there are several, and you find it (and every ones else experiments) to give you a same result, then that is a fact.

Yes - it's a fact that you've produced the same value, if that's what you've done, but in all those cases you're building an assumption of being stationary into your experiment which your experiment is then handing back to you at the end. If you do the experiment based on a different initial assumption, guess what: you get different numbers back for the speed of light relative to the apparatus, but you've completely ignored that.

Quote
If you really expect yourself to have a experiment proving different speeds of light you should construct it and then present your results.

I've already pointed to Michelson-Gale-Pearson which does exactly that.
Title: Re: Can we measure the one way speed of light?
Post by: yor_on on 02/06/2018 14:58:41
what you're doing there David is to go out from a 'whole undividable universe'  in where objects must have a defined speed. Relativity does not do that. You can do it as a presumption, but then you also need to refute the experiments relativity use, at least reinterpret them in a way that builds a logic.
=

What's funny about relativity is that almost everyone I've seen do the same, defending it. And it comes from a presumption, not a fact. What Relativity state is that it's observer dependent, it's not a 'whole undividable universe' in that sense at all
Title: Re: Can we measure the one way speed of light?
Post by: guest4091 on 02/06/2018 17:42:51
The coordinate transformations reduce to
x’ = γx, and t’ =γt.
Thus speed = distance/time.
If x/t = c, then x’/t’ = x/t = c
The reason, motion alters perception and measurement.
This satisfies the 1st postulate of SR since the expressions using x and t are the same for all inertial frames.
Title: Re: Can we measure the one way speed of light?
Post by: alancalverd on 02/06/2018 18:14:19
Yes - it's a fact that you've produced the same value, if that's what you've done, but in all those cases you're building an assumption of being stationary into your experiment which your experiment is then handing back to you at the end. If you do the experiment based on a different initial assumption, guess what: you get different numbers back for the speed of light relative to the apparatus, but you've completely ignored that.
No. Any terrestrial experiment is conducted with the certain knowledge that we are rotating at about 0.25 degrees per second whilst orbiting the sun at godknowshowmany miles per hour and hurtling away from the center of the galaxy at a significant fraction of c whilst other galaxies are rushing about even faster.

Stationary with respect to what?

And when we bounce lasers off the moon, or radar waves off Venus, no part of the apparatus is stationary with respect to any other, but the value of c remains remarkably constant.

Come to think of it, we can measure the doppler shift of a radar or laser reflection from a body moving in a circle if we are standing outside the circle.  Now the reflector has no idea where we are but we know its radius is fixed, and so is the distance from the observer to the center of rotation, so we can calculate its speed relative to the observer in any direction at any time. Lo and behold, the doppler equation applies exactly and everywhere. So c must be constant regardless of direction.
Title: Re: Can we measure the one way speed of light?
Post by: jeffreyH on 02/06/2018 19:13:29
We are not in an inertial frame. As Alan described above. A non inertial frame is actually better for the determination of the one way speed of light. From my perspective this has been put to rest.
Title: Re: Can we measure the one way speed of light?
Post by: yor_on on 02/06/2018 19:14:47
David

" The outcome of the experiment was that the angular velocity of the Earth as measured by astronomy was confirmed to within measuring accuracy. The ring interferometer of the Michelson-Gale experiment was not calibrated by comparison with an outside reference (which was not possible, because the setup was fixed to the Earth). From its design it could be deduced where the central interference fringe ought to be if there would be zero shift. The measured shift was 230 parts in 1000, with an accuracy of 5 parts in 1000. The predicted shift was 237 parts in 1000. According to Michelson/Gale, the experiment is compatible with both the idea of a stationary ether and special relativity.

As it was already pointed out by Michelson in 1904, a positive result in such experiments contradicts the hypothesis of complete aether drag. On the other hand, the stationary ether concept is in agreement with this result, yet it contradicts (with the exception of Lorentz's ether) the Michelson-Morley experiment[citation needed]. Thus special relativity is the only theory which explains both experiments.[6] The experiment is consistent with relativity for the same reason as all other Sagnac type experiments (see Sagnac effect). That is, rotation is absolute in special relativity, because there is no inertial frame of reference in which the whole device is at rest during the complete process of rotation, thus the light paths of the two rays are different in all of those frames, consequently a positive result must occur. It's also possible to define rotating frames in special relativity (Born coordinates), yet in those frames the speed of light is not constant in extended areas any more, thus also in this view a positive result must occur. Today, Sagnac type effects due to Earth's rotation are routinely incorporated into GPS"

https://en.wikipedia.org/wiki/Michelson%E2%80%93Gale%E2%80%93Pearson_experiment
Title: Re: Can we measure the one way speed of light?
Post by: yor_on on 02/06/2018 19:18:45
Phyti, expand on what you write
Title: Re: Can we measure the one way speed of light?
Post by: David Cooper on 02/06/2018 21:01:41
what you're doing there David is to go out from a 'whole undividable universe'  in where objects must have a defined speed. Relativity does not do that. You can do it as a presumption, but then you also need to refute the experiments relativity use, at least reinterpret them in a way that builds a logic.

What I do is consider all possible speeds of an object from rest to just under c - I don't just make an assumption of a single speed for it because there's no way of knowing its actual speed. To consider all possible cases, it's sufficient to look at a few well spaced examples as you can predict what will happen for any other speeds by looking at the pattern you've built up. That means you try out a number of different frames of reference and generate the numbers for everything involved in a scenario. In one frame, object A might be at rest with light moving at c relative to it while object B is not at rest and has light moving in most directions at speeds other than c relative to it. In another frame, object B may be at rest and A moving. In other frames, neither object is at rest. The experiments all fit with this. What you should not do is change frame in the middle of an analysis and say that light is moving at c relative to both objects A and B in all directions.

Quote
What's funny about relativity is that almost everyone I've seen do the same, defending it. And it comes from a presumption, not a fact. What Relativity state is that it's observer dependent, it's not a 'whole undividable universe' in that sense at all

There are two relativity models involved: Lorentz's model and Einstein's (with a number of variants). What relativity universally states is that the analysis from any frame is as useful as any other, producing numbers that are potentially right. I'm not clear as to what your "whole undividable universe" is meant to be, but what MGP shows is that there must be an absolute frame, and our inability to pin that frame down doesn't negate its existence.
Title: Re: Can we measure the one way speed of light?
Post by: David Cooper on 02/06/2018 21:08:19
No. Any terrestrial experiment is conducted with the certain knowledge that we are rotating at about 0.25 degrees per second whilst orbiting the sun at godknowshowmany miles per hour and hurtling away from the center of the galaxy at a significant fraction of c whilst other galaxies are rushing about even faster.

No terrestrial experiment is conducted in the knowledge that we're moving at all at any specific point in time, but over the course of a rotation or orbit, we can be sure that we have been moving most of the time.

Quote
Stationary with respect to what?

The fabric of space which limits the speed of light through it to c.

Quote
And when we bounce lasers off the moon, or radar waves off Venus, no part of the apparatus is stationary with respect to any other, but the value of c remains remarkably constant.

Of course it does - that's exactly what LET says it is, but c is not the one-way speed of light relative to the apparatus.

Quote
Come to think of it, we can measure the doppler shift of a radar or laser reflection from a body moving in a circle if we are standing outside the circle.  Now the reflector has no idea where we are but we know its radius is fixed, and so is the distance from the observer to the center of rotation, so we can calculate its speed relative to the observer in any direction at any time. Lo and behold, the doppler equation applies exactly and everywhere. So c must be constant regardless of direction.

Great: c is a constant, but it is not the one-way speed of light relative to the apparatus.
Title: Re: Can we measure the one way speed of light?
Post by: David Cooper on 02/06/2018 21:15:46
We are not in an inertial frame. As Alan described above. A non inertial frame is actually better for the determination of the one way speed of light. From my perspective this has been put to rest.

Wherever there is a non-inertial frame, there are an infinite number of inertial frames occupying the same location in space, so whatever action you're analysing there must be possible to analyse using any one of those frames and not just your non-inertial frame. If you rule that out, you aren't doing physics any more.
Title: Re: Can we measure the one way speed of light?
Post by: David Cooper on 02/06/2018 21:30:52
According to Michelson/Gale, the experiment is compatible with both the idea of a stationary ether and special relativity.

It is compatible with the predictions of both, but it also disproves SR.

Quote
As it was already pointed out by Michelson in 1904, a positive result in such experiments contradicts the hypothesis of complete aether drag. On the other hand, the stationary ether concept is in agreement with this result, yet it contradicts (with the exception of Lorentz's ether) the Michelson-Morley experiment[citation needed]. Thus special relativity is the only theory which explains both experiments.[6]

Did you notice the words in brackets saying "with the exception of Lorentz's ether"? What that tells you is that LET is compatible with both, so when it goes on to say, "Thus special relativity is the only theory which explains both experiments", it's incorrect. It should say, "Thus special relativity and LET are the only theories which explain both experiments". The article needs editing to correct that, but intelligent people should be able to read the truth out of it regardless by noticing the contradiction and making the correction automatically for themselves.

Quote
The experiment is consistent with relativity for the same reason as all other Sagnac type experiments (see Sagnac effect). That is, rotation is absolute in special relativity, because there is no inertial frame of reference in which the whole device is at rest during the complete process of rotation, thus the light paths of the two rays are different in all of those frames, consequently a positive result must occur. It's also possible to define rotating frames in special relativity (Born coordinates), yet in those frames the speed of light is not constant in extended areas any more, thus also in this view a positive result must occur. Today, Sagnac type effects due to Earth's rotation are routinely incorporated into GPS"

And what I've shown is that there is no bar to exploring the whole action from an inertial frame of reference, and that the results of that analysis must not only be valid, but show in every case (using any inertial frame of reference that you choose) that the red light passes through every part of the ring at a higher speed relative to it than the blue light, which means that the one-way speed of light must be different relative to each part when the light is passing through it depending on whether it's red or blue. No amount of applying bogus rules for non-inertial frames can overturn that fact.
Title: Re: Can we measure the one way speed of light?
Post by: alancalverd on 02/06/2018 22:54:33
No terrestrial experiment is conducted in the knowledge that we're moving at all at any specific point in time, but over the course of a rotation or orbit, we can be sure that we have been moving most of the time.
The idea that the earth moves in jerks is beneath contempt.

Quote
Quote
Stationary with respect to what?

The fabric of space which limits the speed of light through it to c.
So every particle in the universe is dragging the "fabric of space" along with it? So there must be tears in the fabric when two particles move relative to one another. This is beyond bizarre.Whatever happened to Occam?

Quote
Quote
And when we bounce lasers off the moon, or radar waves off Venus, no part of the apparatus is stationary with respect to any other, but the value of c remains remarkably constant.

Of course it does - that's exactly what LET says it is, but c is not the one-way speed of light relative to the apparatus.
The apparatus is a mirror on the moon, or the atmosphere of Venus. Now these move around in space, so if the speed of light depended on direction, we would get different results for the two-way speed depending on the astronomical coordinates of the target.

Quote
Quote
Come to think of it, we can measure the doppler shift of a radar or laser reflection from a body moving in a circle if we are standing outside the circle.  Now the reflector has no idea where we are but we know its radius is fixed, and so is the distance from the observer to the center of rotation, so we can calculate its speed relative to the observer in any direction at any time. Lo and behold, the doppler equation applies exactly and everywhere. So c must be constant regardless of direction.

Great: c is a constant, but it is not the one-way speed of light relative to the apparatus.
[/quote] I can't think of any other definition of what we have measured. If c is constant, then the one-way speed must be the same as the two-way speed, or it wouldn't be "constant".

Never mind. Here's how we measure one-way speed directly.

Make two spinning discs A, B with equispaced circumferential holes. Place them as far apart as you like. Spin A at a constant speed. Shine a light through the holes. Spin the disc B, observe A through the holes of B,  and gradually increase the speed of rotation. The light will dim, brighten, and dim again. The difference in rotational speed between maxima depends on the spacing of the holes and the one-way speed of light. I will leave it to you to write down the equation.
Title: Re: Can we measure the one way speed of light?
Post by: David Cooper on 03/06/2018 19:35:23
No terrestrial experiment is conducted in the knowledge that we're moving at all at any specific point in time, but over the course of a rotation or orbit, we can be sure that we have been moving most of the time.
The idea that the earth moves in jerks is beneath contempt.

There are many ideas that you could introduce here and describe as beneath contempt, but it's normal in a discussion to keep it on topic and not drag extraneous garbage into them, and particularly not to do so in any way that implies that the person you're replying to believes in the random idea that you've flung at them as if it's theirs.

Quote
Quote
Quote
Stationary with respect to what?

The fabric of space which limits the speed of light through it to c.
So every particle in the universe is dragging the "fabric of space" along with it? So there must be tears in the fabric when two particles move relative to one another. This is beyond bizarre.Whatever happened to Occam?

Again, you're dragging in random ideas that have nothing to do with the price of fish. Why would the fabric be dragged by anything? LET doesn't have dragging of the space fabric at all. You clearly don't bother to read anything carefully if you don't already believe in it, or you fail to understand what you're reading because you don't bother to switch your brain on if you know that it goes against your existing beliefs, so you're permanently stuck in the same place, arguing against a theory which you evidently don't understand at all.

Quote
The apparatus is a mirror on the moon, or the atmosphere of Venus. Now these move around in space, so if the speed of light depended on direction, we would get different results for the two-way speed depending on the astronomical coordinates of the target.

According to which theory? Not LET. You've still never crunched the numbers by LET's rules to see what happens with these experiments and how they always produce the same results regardless of the difference of the one-way speeds of light relative to the apparatus in different directions.

Quote
I can't think of any other definition of what we have measured. If c is constant, then the one-way speed must be the same as the two-way speed, or it wouldn't be "constant".

You are either measuring the 2-way speed of light, or you're measuring the effective 2-way speed due to the synchronisation of your clocks hiding the difference. A basic understanding of LET would be all it takes for you to understand this stuff, but you've never bothered to study it at all.

Quote
Never mind. Here's how we measure one-way speed directly.

Make two spinning discs A, B with equispaced circumferential holes. Place them as far apart as you like. Spin A at a constant speed. Shine a light through the holes. Spin the disc B, observe A through the holes of B,  and gradually increase the speed of rotation. The light will dim, brighten, and dim again. The difference in rotational speed between maxima depends on the spacing of the holes and the one-way speed of light. I will leave it to you to write down the equation.

We've covered all of this before - you still don't understand the twist issue as the actual rotation of the leading disc lags behind that of the trailing disc if the apparatus is moving through space - this automatically adjusts the timings to let light pass through the same pair of holes for the same rotation speeds despite the change in the one-way speed of light through the apparatus. But you didn't take that in last time, and there's even less chance of you taking it in now because you're too stuck in your ways; happy to remain ignorant of the facts and just go on repeating bad science. The top experts in SR don't agree with you either, because they understand that LET fits the same facts with all the experiments.
Title: Re: Can we measure the one way speed of light?
Post by: guest4091 on 05/06/2018 18:22:55
Here is a simple scenario with the U reference observing Al moving at .5c along +x axis.
On the left, at the origin, Al sends a light pulse at Ut=0, it reflects at Ut=t, and is detected by Al at Ut=t1. Since the A frame experiences td, Al thinks detection occurs at At=2t', event D'.
On the right, he has no means of determining when the reflection event occurs, but relies on the SR simultaneity convention, and assigns R' to half the round trip time.
If light speed relative to Al is used, as described by U, the reflection event occurs at R''.
This example demonstrates that it makes no difference which method is used. The round trip times are equal, the relative light speeds cv can only be known to an external  (relative to A) observer. Al's measurements support his perception of being in a 'rest' frame.
Any argument for absolute or relative speed of light is just fog.
"That light requires the same time to traverse the same path A to M as for the path B to M is in reality neither a supposition nor a hypothesis about the physical nature of light, but a stipulation which I can make of my own freewill in order to arrive at a definition of simultaneity."
Relativity The Special and the General Theory
Albert Einstein 1961 Crown Publishers Inc. pg 23
 [ Invalid Attachment ]
Title: Re: Can we measure the one way speed of light?
Post by: guest4091 on 05/06/2018 18:25:03
Phyti, expand on what you write
What do you want clarified?
Title: Re: Can we measure the one way speed of light?
Post by: yor_on on 06/06/2018 05:24:22
I was thinking of this one

" The coordinate transformations reduce to
x’ = γx, and t’ =γt.
Thus speed = distance/time.
If x/t = c, then x’/t’ = x/t = c
The reason, motion alters perception and measurement.
This satisfies the 1st postulate of SR since the expressions using x and t are the same for all inertial frames."

You presume too much from us :)
For example you mentioning the "1st postulate of SR"
Title: Re: Can we measure the one way speed of light?
Post by: alancalverd on 06/06/2018 09:13:13
QuoteNever mind. Here's how we measure one-way speed directly.Make two spinning discs A, B with equispaced circumferential holes. Place them as far apart as you like. Spin A at a constant speed. Shine a light through the holes. Spin the disc B, observe A through the holes of B,  and gradually increase the speed of rotation. The light will dim, brighten, and dim again. The difference in rotational speed between maxima depends on the spacing of the holes and the one-way speed of light. I will leave it to you to write down the equation.We've covered all of this before - you still don't understand the twist issue as the actual rotation of the leading disc lags behind that of the trailing disc if the apparatus is moving through space - this automatically adjusts the timings to let light pass through the same pair of holes for the same rotation speeds despite the change in the one-way speed of light through the apparatus. But you didn't take that in last time, and there's even less chance of you taking it in now because you're too stuck in your ways; happy to remain ignorant of the facts and just go on repeating bad science. The top experts in SR don't agree with you either, because they understand that LET fits the same facts with all the experiments.

How does A know the rotational speed of B? How is the distance AB affected by the rotational speed of B? The experiment does not rely on any prior synchronisation, only that the speed of A and the distance AB remain constant.
Title: Re: Can we measure the one way speed of light?
Post by: David Cooper on 06/06/2018 21:12:53
Quote
Quote
Never mind. Here's how we measure one-way speed directly.Make two spinning discs A, B with equispaced circumferential holes. Place them as far apart as you like. Spin A at a constant speed. Shine a light through the holes. Spin the disc B, observe A through the holes of B,  and gradually increase the speed of rotation. The light will dim, brighten, and dim again. The difference in rotational speed between maxima depends on the spacing of the holes and the one-way speed of light. I will leave it to you to write down the equation.

We've covered all of this before - you still don't understand the twist issue as the actual rotation of the leading disc lags behind that of the trailing disc if the apparatus is moving through space - this automatically adjusts the timings to let light pass through the same pair of holes for the same rotation speeds despite the change in the one-way speed of light through the apparatus. But you didn't take that in last time, and there's even less chance of you taking it in now because you're too stuck in your ways; happy to remain ignorant of the facts and just go on repeating bad science. The top experts in SR don't agree with you either, because they understand that LET fits the same facts with all the experiments.

How does A know the rotational speed of B? How is the distance AB affected by the rotational speed of B? The experiment does not rely on any prior synchronisation, only that the speed of A and the distance AB remain constant.

The way you've described the experiment is deficient, so I don't know how you envisage the one-way speed of light having any relevance to it - you can increase the distance between A and B without noticing the behaviour changing unless you're also making precise timings and introducing them into it in some way that you haven't specified. For my previous reply, I assumed additional requirements of the experiment which you hadn't provided, and twist (synchronisation issues) will always hide the one-way speed of light from you. Without a better description of your experiment though, and if I'm to take it exactly as you've set it out, the distance AB can be set to any length you like without it changing the result of the experiment.
Title: Re: Can we measure the one way speed of light?
Post by: David Cooper on 06/06/2018 21:24:41
Here is a simple scenario with the U reference observing Al moving at .5c along +x axis.

It may be simple, but I can't map the description to the diagram. It would be a lot easier to follow these things if you'd describe the scenario in ordinary words and if the things in the description also exist in the diagram.

Quote
Any argument for absolute or relative speed of light is just fog.

On the contrary, looking at the actual reality with the speed of light varying relative to the experiment is where you clear the fog and look at what might actually be going on instead of fooling yourself into thinking every experiment you do is at rest.
Title: Re: Can we measure the one way speed of light?
Post by: alancalverd on 06/06/2018 22:54:22
David: you are right. My experiment is wrong.

Not that it matters,since c = 1/√ε0μ0 so it is independent of direction anyway, as ε and μ are scalar quantities.
Title: Re: Can we measure the one way speed of light?
Post by: yor_on on 06/06/2018 23:29:47
Alan, you can't use rotating discs for it
Title: Re: Can we measure the one way speed of light?
Post by: guest4091 on 07/06/2018 16:22:22
I was thinking of this one

" The coordinate transformations reduce to
x’ = γx, and t’ =γt.
Thus speed = distance/time.
If x/t = c, then x’/t’ = x/t = c
The reason, motion alters perception and measurement.
This satisfies the 1st postulate of SR since the expressions using x and t are the same for all inertial frames."

You presume too much from us :)
For example you mentioning the "1st postulate of SR"
I thought everyone participating in these 'relativity' subjects was familiar with the 1st and 2nd postulates of SR.
The 1st: the description/equations of physics is the same in any inertial frame.
Example:
The expression x=vt should work in any frame.
With light speed =1, on Earth, Al calculates the distance of object D as x=vt/2=1*8/2 =4 lyr. ( He needs a return signal)
In a spacecraft moving at .5, past Earth, toward D, he calculates x=.5*7=3.5 yrs.
As an anaut in a pseudo rest frame, he has to allow for length contraction of the universe as it moves past him at .5 in the opposite direction.
Does the moving anaut cause the universe to contract by moving extremely fast? No.
Do the particles in an accelerator cause the universe to contract by moving at near light speed? No.
This is actually his perception or interpretation of events, to reconcile the effects of time dilation.
On Earth D was 4 ly distant. Now, in his craft, it's arrived early, according to his clock (biological and mechanical). He can't deny his time nor his speed, so the only thing left is distance. If he sent a signal to D as he passed Earth, his conclusion would be the same, lc, i.e. verified by measurement.
The formula does not require any modifications.
In a world of absolute motions, you would have to know the location of the center of mass or absolute reference frame,) which is impossible in a dynamic universe. That's the beauty and utility of Relativity.
Imagine if you could have a conversation with someone, anywhere on Earth without requiring a translation.
Title: Re: Can we measure the one way speed of light?
Post by: guest4091 on 07/06/2018 20:21:39
David Cooper#53;
Quote
Never mind. Here's how we measure one-way speed directly.Make two spinning discs A, B with equispaced circumferential holes. Place them as far apart as you like. Spin A at a constant speed. Shine a light through the holes. Spin the disc B, observe A through the holes of B,  and gradually increase the speed of rotation. The light will dim, brighten, and dim again. The difference in rotational speed between maxima depends on the spacing of the holes and the one-way speed of light. I will leave it to you to write down the equation.
Who is measuring the light transit time?
Title: Re: Can we measure the one way speed of light?
Post by: jeffreyH on 07/06/2018 20:59:18
@phyti You could actually come down off your high horse and help others.
https://en.m.wikipedia.org/wiki/Postulates_of_special_relativity
Title: Re: Can we measure the one way speed of light?
Post by: guest4091 on 08/06/2018 19:07:18
It may be simple, but I can't map the description to the diagram. It would be a lot easier to follow these things if you'd describe the scenario in ordinary words and if the things in the description also exist in the diagram.
Here's the new and improved version.
Fig.1 is the U frame description.
A moving at .5c, sends a signal (blue) to mirror M at Ux=2.00. The signal reflects at R, and is detected by A at Ut=2.67. U observes the A-clock running slow. The hyperbola (red) is a line of constant time, and transfers At to the Ut scale.
Fig.2 is the A frame description.
M is moving at .5c in the -x direction, reflects the A signal, which A detects at At=2.31.
A assigns the time of reflection to R', per the SR simultaneity convention (half of round trip time). A calculates the distance to M as 1.73 at At=0, i.e. length contracted.
In fig.1, U observes light speed relative to A as .50 outbound and 1.50 inbound. The path 0RD is overlaid in fig.2 for comparison. The round trip time is the same for A.
This is the justification for Einstein's simultaneity definition, with equal path lengths out and back. Since there is no means or need of determining the absolute A-time for the reflection, consistency of the theory is the priority.
Using the coordinate transforms CT:
x' =  γ(x-vt) = 1.15[2.00 - .5(2.00)] = 1.15
t' =  γ(t-vx) = 1.15[2.00 - .5(2.00)] = 1.15
A check to support the fact that the spacetime graphics are a geometrical  representation of the CT.
A 2nd example is shown with the event off the diagonal.
x' =  γ(x-vt) = 1.15[1.50 - .5(1.00)] = 1.15
t' =  γ(t-vx) = 1.15[1.00 - .5(1.50)] = .29
 [ Invalid Attachment ]  [ Invalid Attachment ]
Title: Re: Can we measure the one way speed of light?
Post by: guest4091 on 08/06/2018 19:23:12
@phyti You could actually come down off your high horse and help others.
https://en.m.wikipedia.org/wiki/Postulates_of_special_relativity

I don't ride horses!.
If you want more references, I can do that.
I'll repeat, that most here seem to know the basics of SR, and some like pmb, know the whole theory.
I'll also admit that someone who works with an idea more than others, may forget that others need more detail for understanding. I may not agree with other opinions, but I don't insult them, just offer an alternate explanation.. My advice is the standard, "if you don't understand, ask".
Title: Re: Can we measure the one way speed of light?
Post by: yor_on on 08/06/2018 22:08:15
Phyti , I kind of like you, but I also think you hide. What is more is that you present it in such a way that it makes little sense to those not having a background in the mathematics you push for. Neither of those things impress me
=

don't get mad now :)
The point I'm making is that if you want to push for something you think someone else is arguing about, better ask first, and then define the terms you're using to define where they go wrong. Keep it clear and don't overwhelm the reader. It's at that time you lose them. If you're as good as you think you are, you should be able to do this
Title: Re: Can we measure the one way speed of light?
Post by: yor_on on 08/06/2018 22:34:12
I know, for you the mathematics speaks :)
But it doesn't for all of us Phyti

so simplify, find that reason why the mathematics must hold. and then describe it
Title: Re: Can we measure the one way speed of light?
Post by: David Cooper on 08/06/2018 23:41:20
Here's the new and improved version.

Much better - thanks.

Quote
This is the justification for Einstein's simultaneity definition, with equal path lengths out and back. Since there is no means or need of determining the absolute A-time for the reflection, consistency of the theory is the priority.

It's simply assuming that A is at rest and that the two light paths are of equal length, so the real justification for that synchronisation is that light is taken to be travelling at c relative to A. In the first case, where you took M as being at rest instead, the speed of light relative to A was not c. If you put a mirror with A and bounce the signal back to M, you can justify imagining that M is at rest when thinking things through for someone travelling with M on the same basis of assuming that both light paths (this time the ones starting from and then coming back to M) are of equal length. You thus have two contradictory accounts of events (one for A and one for M) which cannot both be true accounts of the action, but either one of them could be true, and no one will ever know. The inability to tell what speed light is actually going at relative to either A or M is not justification for claiming that there is no single right answer. The only way you can make the speed of light relative to both A and M equal to c is by changing reference frame in between looking at them, and you're then failing to recognise that you're having your cake and eating it by changing frame between measurements rather than sticking to a single inertial (i.e. genuine) frame for both. Using the frame in which A is at rest automatically asserts that the speed of light relative to M is not c, and using the frame in which M is at rest automatically asserts that the speed of light relative to A is not c, but what do you do: you selectively prune out all the things that aren't moving at c relative to light and deny that those relative speeds (that aren't c) count for anything, asserting that you have to change frame until they are moving at c relative to the light before that relative speed magically becomes the valid one for them, and in this way you make the one-way speed of light c relative to all objects.

Let's play the same kind of silly game with light itself. In some frames of reference, some photons are moving at 2c relative to other photons, but others are moving in the same direction with a relative speed of zero. What about all the values in between? Well, if they aren't moving on parallel paths we can always select a frame of reference in which we can assert that they're moving apart at 2c, so we can play the frame-switching game and make the claim that light always travels at 2c relative to other light unless it's co-moving, and any case where that doesn't appear to be the case is irrelevant because we should only analyse it by switching to a frame which makes it a relative speed of 2c. Only frauds would play such a game.
Title: Re: Can we measure the one way speed of light?
Post by: alancalverd on 09/06/2018 07:56:12
Alan, you can't use rotating discs for it
Agreed. See my reply #55, which also points out that as c is a scalar, not a vector quantity, it is independent of any direction, so the one-way speed must equal the two-way speed.
Title: Re: Can we measure the one way speed of light?
Post by: jeffreyH on 09/06/2018 09:39:27
Why is c a scalar? What are the historic reasons? Is it because it is considered a constant? A vector can certainly be defined for a photon.
Title: Re: Can we measure the one way speed of light?
Post by: Colin2B on 09/06/2018 10:44:57
Why is c a scalar? What are the historic reasons? Is it because it is considered a constant? A vector can certainly be defined for a photon.
Because it is the general case before a direction is defined? So just the magnitude.
Title: Re: Can we measure the one way speed of light?
Post by: jeffreyH on 09/06/2018 15:22:16
Yes for a locally measured magnitude.
http://math.ucr.edu/home/baez/physics/Relativity/SpeedOfLight/speed_of_light.html
Title: Re: Can we measure the one way speed of light?
Post by: alancalverd on 10/06/2018 11:37:30
No.

ε0 and μ0 are measured in entirely separate, static experiments.

If the value of c depended on direction, at least one of these would have to be a vector so you would always get different values for either ε0 or μ0 depending on the polarity of the applied electric or magnetic field. You don't.

For anyone with a real interest in experimental physics it is worth noting that the relative values of εxyzand μxyz for some materials are indeed anisotropic tensors and the magnitude of c does indeed vary with the orientation of the material. The fact that the resulting birefringence effects are independent of the orientation of the material with respect to the fixed stars, phase of the moon, or anything else, strongly suggests that there is no aether or other universal frame of reference, so c0 is independent of direction.
Title: Re: Can we measure the one way speed of light?
Post by: David Cooper on 10/06/2018 22:40:35
ε0 and μ0 are measured in entirely separate, static experiments.

They aren't static if the experiments are moving. How are you calculating them? Are you using any distance measurements? If so (clue: you are), then you are supplying those measurements on the basis that the experiment is stationary, even though it may not be. You therefore fool yourself into imagining that there's no aether because you only have a simplistic understanding of what you're doing. It is the phenomenon of relativity itself that hides all the differences from you - that's precisely what allows you to carry out experiments on a naive basis and still get answers that work.
Title: Re: Can we measure the one way speed of light?
Post by: guest4091 on 11/06/2018 16:40:53
Yoron#62;
Thanks for sharing your thoughts.
I do think if many words do not impart understanding, your effort is wasted. I remind myself to keep it simple and clear for the reader, but am not always successful. When reviewing papers I've written in the past, if they seem faulty, they go in the trash. I'm my worst critic, so it's not a question of 'how good you think you are', but how well you convey understanding to others. I'll work to improve.
Title: Re: Can we measure the one way speed of light?
Post by: guest4091 on 11/06/2018 16:44:34
David C #64;
The 1st observer is actually U, and M is in his frame. Fig.1 is drawn according to his description/perception of events. The U frame is randomly selected from a multitude of frames with a velocity different from A, i.e. with a relative motion. U is not special in any way. The light path 0RD is associated with U only.
Returning to the 'train' example, the passenger on the moving train extends his hand forward and releases a stone. The path he sees is a straight vertical line to the floor. For  the bystander on the platform, the path is a curved line from the hand to the floor.
The path, the planetary orbits, missile trajectories, and even the light path, are histories of position, that reside in the mind, and have no physical objectivity outside the mind. As it is with the train example, so it is with the reflection example. The perception depends on the motion of the observer. 
In fig.2, A cannot observe himself as in an 'out of body experience'. A is present at At=0 and At=2.31, but not at the remote reflection. A does not know the local time of the reflection. As Einstein stated, it's not about the propagation of light, but a simple solution that resolves the issue of the remote event while in agreement with the constant c.
I know you are an advocate of LET from previous exchanges.
I added the relative light speeds, .5c outbound and 1.5c inbound, from U's perception to show, there is no difference. If you have read the 1905 paper, you would be aware that Einstein used relative speeds c+v and c-v to develop SR, which implies an absolute reference frame as an initial assumption, but disappears when the theory is simplified to one depending only on the relative speeds of the observers.
Lack of math skills or graphical interpretations will be a handicap for some.
There is nothing contradictory about two observers in relative motion having equivalent perceptions, such as reciprocal time dilation, or simultaneously different perceptions of an object in motion. It's not just about matter, light and motion. It's about knowing the difference between objects in motion and images of objects as processed by the mind.
SR was never presented as a theory of perception by it's author or those who modified it, but it should be, since it has the qualifications.
Title: Re: Can we measure the one way speed of light?
Post by: David Cooper on 11/06/2018 21:09:53
I added the relative light speeds, .5c outbound and 1.5c inbound, from U's perception to show, there is no difference.

No difference to what? Only to the illusion that each observer is stationary.

Quote
If you have read the 1905 paper, you would be aware that Einstein used relative speeds c+v and c-v to develop SR, which implies an absolute reference frame as an initial assumption, but disappears when the theory is simplified to one depending only on the relative speeds of the observers.

It doesn't disappear because the attempt to remove it leads to an infinite number of contradictions as soon as you assert that there is no absolute frame.

Quote
Lack of math skills or graphical interpretations will be a handicap for some.

Indeed, and the acceptance of contradictions is one of the worst failures you can make in maths.

Quote
There is nothing contradictory about two observers in relative motion having equivalent perceptions

There is something contradictory though when you assert that both accounts of events are true in terms of what has actually happened. Things would be fine if this stuff was just being pushed as a theory of perception, but it isn't - it's being pushed as an absolute account of reality in which contradictions are generated by the business of having one's cake and eating it.
Title: Re: Can we measure the one way speed of light?
Post by: jeffreyH on 11/06/2018 21:16:30
No.

ε0 and μ0 are measured in entirely separate, static experiments.

If the value of c depended on direction, at least one of these would have to be a vector so you would always get different values for either ε0 or μ0 depending on the polarity of the applied electric or magnetic field. You don't.

For anyone with a real interest in experimental physics it is worth noting that the relative values of εxyzand μxyz for some materials are indeed anisotropic tensors and the magnitude of c does indeed vary with the orientation of the material. The fact that the resulting birefringence effects are independent of the orientation of the material with respect to the fixed stars, phase of the moon, or anything else, strongly suggests that there is no aether or other universal frame of reference, so c0 is independent of direction.

This is exactly why this is the best science forum. Is everyone taking this in? This is a very unique opportunity to learn. Don't waste it. No tuition fees!
Title: Re: Can we measure the one way speed of light?
Post by: guest4091 on 13/06/2018 16:08:48
David C.#73;
Quote
There is something contradictory though when you assert that both accounts of events are true in terms of what has actually happened. Things would be fine if this stuff was just being pushed as a theory of perception, but it isn't - it's being pushed as an absolute account of reality in which contradictions are generated by the business of having one's cake and eating it.

If 10 people form a circle around a house, each will have a different view of the house.
There are 10 simultaneous views, each different from the others.
Who has the true/real view if there is only one house?
This simplest of cases shows the question is meaningless. If there were x people, all x  views would be correct, relative to each person. The statements are true concerning the images, but each image is incomplete regarding the house. All the images collectively are incomplete since they only provide information about the exterior of the house. The statements are about perception, not about the house.

In science, information is verified by measurement. Experiment has demonstrated that motion affects measurement and perception. There is no expectation that two observers moving at different velocities will make identical measurements or receive identical images of events.
In the last example, U makes measurements from his location and assigns local times to events. In the U frame, A is moving at v and light is moving at c.
A makes measurements from her location and assigns local times to events. In the A frame, U is moving at v and light is moving at c.
Each can say light is moving at c-v or c+v relative to the other, but not relative to themselves. They have no means of that type of measurement.
All an observer has is measurements and perception.
 
"it's being pushed as an absolute account of reality"

FALSE, since 'RELATIVITY' makes no such claim, and has no need for the concept 'absolute'. Why is the 1905 paper titled "On the Electrodynamics of Moving Bodies", and not some variation of "Relativity"?
The common phenomenon of induction was one example that didn't need an absolute reference. Move the magnet or move the coil, there was still current flow. Which object was 'really in motion' was irrelevant.

You miss the point of the principle of relativity.
Time dilation: (the most often misinterpreted)
The 1st postulate states physical processes occur the same way in any inertial frame.
1. If A observes the B-clock moving past him at high speed, it's perceived to run slower than the A-clock.
2. If B observes the A-clock moving past him at high speed, it's perceived to run slower than the B-clock.
It would be a contradiction of postulate-1, if both observing the same process, got different results. It would also be a strange world with no predictability. 
The math (algebra and geometry) consists of equations that are symmetrical with respect to A and B. It's as simple as swapping A and B in the 1st statement to form the 2nd statement.
What you call contradictory is actually reciprocity
Title: Re: Can we measure the one way speed of light?
Post by: David Cooper on 13/06/2018 22:13:01
If 10 people form a circle around a house, each will have a different view of the house.

And each view maps to the same model of the house rather than to ten incompatible models of the house which contradict each other, so your analogy doesn't fit.
 
Quote
Who has the true/real view if there is only one house?

They all map to the same one. In SR, you only have that if you go to an extreme model in which light reduces all the paths it follows to zero length and zero time, and with such a model you get event-meshing failures unless you go to a block universe model, and even if you go there, you get stuck because you can't account for the generation of that block without creating it under different rules (i.e. not SR).

Quote
The statements are about perception, not about the house.

Your analogy doesn't fit, but if it's all about perceptions, let's see SR being discussed in such a way that it only deals with those and stops making incompatible assertions about the actual house.

Quote
"it's being pushed as an absolute account of reality"

FALSE, since 'RELATIVITY' makes no such claim, and has no need for the concept 'absolute'. Why is the 1905 paper titled "On the Electrodynamics of Moving Bodies", and not some variation of "Relativity"?
The common phenomenon of induction was one example that didn't need an absolute reference. Move the magnet or move the coil, there was still current flow. Which object was 'really in motion' was irrelevant.

Stop pretending that SR makes no such claims. SR claims that there is no aether, and in doing so it ceases to be a mere theory of perceptions. We have armies of experts going around laying down the law as to how there is no aether and that the one-way speed of light is always c relative to any object it passes, and they attribute this to SR. SR is a theory of reality. Physics is an attempt to explain what reality is. If you want to take issue with the things people are saying here, you need to go and jump on all the ones who are asserting those things (about the non-existence of aether and the speed of (unslowed) light always being c relative to whatever it passes) and tell them they're out of line, bringing science into disrepute.

Quote
You miss the point of the principle of relativity.

I haven't missed the point at all. You are the one who has missed it, and it's because you're blind to contradiction.

Quote
What you call contradictory is actually reciprocity

No - it's plain contradiction. If its true that light passes A at c relative to A and that it also passes B at c relative to B while A and B are moving relative to each other along the same line, you must have light overtaking itself as it travels between A and B because it's moving faster than itself. In reality though, it is not moving at c relative to both A and B along that line. You've sabotaged your own thinking so extensively that you're incapable of recognising this kind of contradiction. For that reason, it would be best for you to look carefully at MGP instead (where it's harder to hide from the truth) and to let it help you unshackle your mind, but I'll say more about that in my next post because it's aimed at everyone who believes in SR. I want them to stop hiding from the truth by spelling out where they stand on the questions that it raises.
Title: Re: Can we measure the one way speed of light?
Post by: David Cooper on 13/06/2018 22:13:35
A challenge to all those who haven't yet seen the light:-

The Michelson-Gale-Pearson experiment showed what happens when you rotate a circuit and send light round it in opposite directions. I've pointed to this before from this thread, but the SR fans have failed to commit themselves to any pronouncements on it because it proves that the one-way speed of light is not always c in all directions relative to the things it passes and by extension that there must be an absolute frame. It's time for them to stop running away from this and to state their position on each question that I ask them to answer.

Imagine a ring of empty tube lined with a mirror coating. We will label it with a hundred sectors (or a million, or any big finite number you like), each sector being the same size. When we send light round the circuit, it is sent out from S0 (sector zero) and travels either through S1 next or S99 depending on which way it travels round the circuit. We will rotate the ring anticlockwise (as if it's sitting round the Earth's equator with us looking down at it from over the north pole [or it could be in orbit over the equator to make it perfectly circular and so that the tube contains a vacuum rather than air]). We will send pulses of blue light round the circuit in the anticlockwise direction too (S0, S1, S2, etc.), while pulses of red light will be sent clockwise (S0, S99, S98, etc.).

If we rotate the ring at a particular speed, we can observe pulses of blue and red light being emitted simultaneously from S0 in opposite directions and then travelling round the ring thousands of times before arriving back at S0, again simultaneously, after the ring has completed a single rotation. The red light will have passed through S0 (and every other sector of the circuit) one time more than the blue light has.

For each sector, the red light has passed through it at an average speed of R relative to that sector while it was passing through that sector.

For each sector, the blue light has passed through it at an average speed of B relative to that sector while it was passing through that sector.

The same amount of time has passed for each sector, and this can be measured by clocks local to each sector. Each sector has also travelled the same distance, regardless of which frame of reference you use to measure that.

So, is R=B, or is R>B?

If your answer is R>B, you are necessarily agreeing that the average speed of the red light relative to each sector it passes through while it's passing through that sector is >c and that the average speed of the blue light relative to each sector it passes through while it's passing through that sector is <c.

If your answer is R=B, then you need to deny the result of the MGP experiment and assert that the red light cannot have passed through each sector of the circuit an extra time.

Where do you stand? Are you in the R=B camp or the R>B one? Who here has the courage to post their answer to this simple question?

For the record, my answer is R>B, and this is the case for all inertial frames of reference. If you want to try to explain how the problem can produce R=B by using a non-inertial frame instead, feel free to set out your analysis for that, and then I'll show you why it isn't valid. (Clue: every non-inertial frame of reference has to be compatible with an inertial frame so that an inertial observer co-moving with the apparatus doesn't have light moving at different speeds relative to him in the same direction at opposite parts of the circuit.)
Title: Re: Can we measure the one way speed of light?
Post by: guest4091 on 14/06/2018 15:49:56
David Cooper#30;
In fig.1, U describes a fiber-optic cable (ring) at rest, with light emitted in opposite directions from a device at A, which reflects from a mirror M at the other end of the diameter. The signals travel one complete revolution and return simultaneously.

In fig.2, the ring rotates ccw. The cw light arrives at the mirror before the ccw light as seen by U.

With the device A serving the role of moving observer, the center of the ring remains fixed relative to U and A, but A is moving relative to U. The time axis is perpendicular to and centered on the ring.
The light paths are spirals around the time axis, as if on the surface of an invisible tube, 0-R2-D ccw and 0-R1-D cw. For simplicity the tube is unfolded to a flat surface as in fig.3. The minus signs on the x axis indicate small lc. The 3rd blue signal ending at At = 6.28 is the round trip time for both signals.
The round trip times will be equal for A, with a measured constant light speed of c.
Per the equivalence principle, A would experience a weak g-field from the rotation.
Fig.3 represents the description by an observer who is NOT rotating with the ring.
As with the inertial frames, you are imposing the description by U (the rest frame) onto the perception of the moving frame A. Both cannot have the same perception if there is relative motion between them. U may have an advantage of observing all 3 events, 0, R, and D, which are not local for U, and also modified by his motion, which is also relative.
Note a rotating frame is considered absolute motion in Relativity.
 [ Invalid Attachment ]
 [ Invalid Attachment ]
Title: Re: Can we measure the one way speed of light?
Post by: David Cooper on 14/06/2018 23:23:07
In fig.1, U describes a fiber-optic cable (ring) at rest, with light emitted in opposite directions from a device at A, which reflects from a mirror M at the other end of the diameter. The signals travel one complete revolution and return simultaneously.

I don't understand part of your experiment's description. How can the signals travel one complete revolution when they're bounced back at M and don't get to go the rest of the way round? You appear to have designed an experiment where both lots of light go from A to M and then back the way they came, and they both arrive back at A simultaneously precisely because neither of them do a circuit, but they instead do half a circuit each way and spend equal amounts of time going anti clockwise as each other, plus equal amounts of time going clockwise as each other. This avoids the entire issue put to you by my experiment where the two signals don't arrive back at A simultaneously (unless A has completed a full revolution, by which time one lot of light has done an extra circuit compared with the other lot).

Quote
The minus signs on the x axis indicate small lc.

Not sure what you mean by lc. It would be worth explaining now in case this comes up again.

Quote
As with the inertial frames, you are imposing the description by U (the rest frame) onto the perception of the moving frame A.

Not so, and this is not my experiment either, but yours - you have the two lots of light getting back to A simultaneously and thus we are in agreement about A's perception. The question now is whether we're in agreement about S0's perception in my experiment (which is its closest equivalent to your A).

So, what is S0's perception in my experiment? If I'm moving with S0 throughout, what do I see? I see the red light go off westwards and blue go off at the same moment eastwards. The red light comes back from the east and heads out westwards again, then the blue light comes back from the west and heads out eastwards again. This happens many times with the time gap between the returns of the two lots of light growing greater each time, but eventually I see them both arriving simultaneously and note that red has done an extra lap. I determine from this that because the red light has done an extra lap, it must have moved faster on average relative to each sector it passed through, while passing through that sector, faster than the blue light did. I am A in this case (with a non-inertial frame of reference as my measuring standard), and I have just given you A's perception that R>B, just like all the other observers regardless of which inertial or non-inertial frame they're using for their analysis.

So, S0's observer A (in his non-inertial frame) determines that the average one-way speed of light relative to each of the hundred sectors while it's passing through it is greater than c in one direction (anticlockwise) and less than c in the other (clockwise). What kind of voodoo do you have to resort to to turn his perception into something that agrees with SR (given that SR denies the ability of unslowed light to move at any speed relative to anything than c)? The only fix is to move to one of the SR variant models in which light reduces all paths to zero length in zero time, at which point you can hide the problem in an infinity by removing the property of speed from light altogether (though that too has unpalatable consequences for you).

[Note my careful wording: "average speed of light relative to each of the hundred sectors while it's passing through it" - if you ignore the "while it's passing though it" part, you will get a different result which would mask the truth of what's going on. My careful wording is there to force you only to consider the speed of light relative to any sector that it is passing through at the time while it's passing through it and not when the light is elsewhere in the circuit. It is only when you respect this wording that you force the truth out from the experiment.]
Title: Re: Can we measure the one way speed of light?
Post by: guest4091 on 15/06/2018 16:58:39
David c.#76

The house example is only intended to show all observers aren't required to have the same perception or point of view, even without motion.
---------
Quote
and with such a model you get event-meshing failures unless you go to a block universe model,
 
-----
There are no such things as 'event-meshing failures' (your invention).
-------
Quote
Stop pretending that SR makes no such claims. SR claims that there is no aether, and in doing so it ceases to be a mere theory of perceptions.

---------
I don't have to pretend. All experiments to detect any effect of an ether have failed. Einstein called it superfluous and developed SR without it, yielding the same coordinate transformations as developed by Lorentz.
If you understand perception, then you know the brain is a complex process which behaves according to rules of physics, specifically chemistry. That is also 'reality'. Medical science can monitor this activity and 'map' certain areas to a corresponding activity. It's not just inanimate clocks that run slow when moving fast, but also biological clocks.
---------
Quote
one-way speed of light is always c relative to any object it passes
---------
SR does not state that. It does state "the measured speed of light relative to an inertial frame is c. In #72  the relative speed of light was overlaid in the A frame to show the round trip time (2.31) would be the same.
If U and A had the same perception about everything, why have transformations?
------------
Quote
No - it's plain contradiction. If its true that light passes A at c relative to A and that it also passes B at c relative to B while A and B are moving relative to each other along the same line, you must have light overtaking itself as it travels between A and B because it's moving faster than itself.
---------
Your interpretation is flawed. You are not compensating for time dilation and length contraction for both A and B. Those motion induced effects alter perception.
Title: Re: Can we measure the one way speed of light?
Post by: David Cooper on 15/06/2018 22:50:51
The house example is only intended to show all observers aren't required to have the same perception or point of view, even without motion.

Fine, but their perceptions must all map to the same reality. With relativity, the different perceptions can map to many alternative potential realities, only one of which will be the actual reality.

Quote
There are no such things as 'event-meshing failures' (your invention).

Event-meshing failures are an automatic consequence of SR - it isn't any invention of mine (other than the name), but a discovery. If you run a relativity simulation in which the only time allowed in the model is the time of the time dimension, you necessarily get event-meshing failures, and to deny that is to deny mathematics. It is shocking that the SR experts didn't already have their own name for this. I've written a simulation that demonstrates this in action and I've asked anyone who rejects what it shows to produce or point to any simulation of relativity that runs SR by its own rules without producing the same event-meshing failures, but none have been able to do so (because it is mathematically impossible to do so). All they have are simulations that cheat by bringing in an external (Newtonian) time to govern the unfolding of events, which means they're actually running LET with an aether and are dishonestly passing it off as SR.

Quote
I don't have to pretend. All experiments to detect any effect of an ether have failed. Einstein called it superfluous and developed SR without it, yielding the same coordinate transformations as developed by Lorentz.

You've just done it right there: the bit about the ether being superfluous. It isn't - it's vital, and MGP shows that it exists.
 
Quote
It's not just inanimate clocks that run slow when moving fast, but also biological clocks.

Who are you trying to correct here? LET says exactly that.

Quote
SR does not state that. It does state "the measured speed of light relative to an inertial frame is c.

Great, so if it doesn't assert that the one-way speed of light relative to any object is always c and can't be greater than that, you should be stamping down on the people who claim that it is always c, but you never do that, and armies of SR pushers are systematically miseducating the public with the same claims.

Quote
If U and A had the same perception about everything, why have transformations?

The transformations switch between different potential realities. Every time you use one, you are changing the asserted speed of light relative to the content of the system in some directions.

Quote
Your interpretation is flawed. You are not compensating for time dilation and length contraction for both A and B. Those motion induced effects alter perception.

It isn't flawed at all - you're just trying to have your cake and eat it again by switching from one potential reality to another while refusing to recognise that only one underlying reality is the real one.

Again though, what we're seeing is a game of avoidance, not just from you, but from everyone else here. What is your answer to the R>B vs. R=B issue? I've given you a clear experiment that resolves the whole issue by showing that the one-way speed of light relative to an object is >c in one direction and <c in the opposite direction. No one here other than me has had the courage to state an answer to that really simple little question, so what's holding them back?

Here's a list of little questions which everyone should be able to provide yes/no answers to without significant hesitation:-

Q1: If we have an observer X co-moving with S0 throughout, does he see the pulse of red light pass him more often than the blue light? (The correct answer is yes, and it is also yes for all observers - they see the red light pulse passing him more often than the blue one.)

Q2: Does observer X measure the ring as having the same length in both directions from S0 round the ring and back to S0? (The correct answer is yes, and it is also yes for all other observers.)

Q3: Given that the red light returns faster than the blue light each time, should observer X conclude that the red light has passed through the sectors of the ring at a higher average speed relative to them (while it's passing through them) than the blue light? (The correct answer is yes: he should respect mathematics and conclude that R>B.)

Q4: If we have an observer Y next to the rotating ring such that X is with him initially, but then X goes round with the ring next to sector S0 and eventually returns to Y at a moment when the pulses of red and blue light happen to arrive there simultaneously, should observer Y also conclude that the red light has passed through the sectors of the ring at a higher average speed relative to them (while it's passing through them) than the blue light? (The correct answer is yes: he too should respect mathematics and conclude that R>B.)

Q5: Do all observers have a perception which should lead them to conclude that R>B? (The correct answer is yes - all possible observers recognise that the red light has travelled through the 100 sectors at a higher speed relative to them on average than the blue light, assuming that they respect mathematics.)

Anyone who is incapable of answering these questions should ask themselves what's stopping them other than their determination to stick to their existing beliefs instead of adapting to reality.
Title: Re: Can we measure the one way speed of light?
Post by: guest4091 on 16/06/2018 17:58:22
David C.;

blue lines are light paths, light gray are dimensions
Ann is the ref frame. Ben is moving at .6c in the +x direction as he passes Ann.
Ann sends a light signal from the origin in the +x direction as Ben passes.
Ann observes Ben moving at .6/sec and observes* a light signal moving at 1.0/sec.
Ann does not observe anything moving at .4/sec, but concludes the distance s  separating Ben and the signal is expanding at a rate of .4/sec.

Ben has a rod pointing in the x direction, with a mirror m at the far end. The rod s  measures 1 light unit in length. (nano, micro, ..., sec). Ben sends a signal from the origin to m. Ben detects the return at Bt=2.0, due to time dilation and length contraction, indicated in red. Ben calculates light speed as x'/t'=1/1=c. Ben does not observe anything moving at .4/sec.

Moving forward in time from At=0, the space between the near end of the rod and the photon expands at a rate of (1-.6)=.4.
Moving forward in time from At=2.0, the space between the near end of the rod and the photon contracts at a rate of (1+.6)=1.6. 

These cases are about closing speeds, i.e. not speeds of physical objects, but the rate of change for a relationship.

* records the events R and D via additional light from those events, and the assumption that light moves in straight lines between those events, and any needed calculations.
 [ Invalid Attachment ]
Title: Re: Can we measure the one way speed of light?
Post by: David Cooper on 16/06/2018 22:09:34
David C.;

blue lines are light paths, light gray are dimensions...

Lovely, but why spend so much time on something we already agree on? You know that LET produces the same accounts of events to document the same perceptions. Where we differ is that you deny the role of the absolute frame, and you have to tolerate contradictions as a consequence of that denial (though you've been programmed to be blind to those contradictions through miseducation). If Ann is really stationary, her account represents reality while Ben's account is a distorted view of events. If Ben is really stationary, his account represents reality while Ann's account is a distorted view of events. If neither are stationary, both their accounts are distorted views of events.

My experiment, which you are still failing to address, demonstrates that there is a difference in the actual speed of light relative to the material of the ring in opposite directions as it travels through the sectors. The closing speed (seeing as you understand that term) between the red light and the material of a sector it's about to travel through next must on occasions be greater than c - we just can't pin down where in the ring this applies, so we can't identify the absolute frame from this. However, we can tell that there must be an absolute frame, because we clearly must have one at any location where the speed of light is greater than c in one direction relative to the ring material there and less than c in the opposite direction relative to that same material, and that absolute frame would necessarily have that material moving through it rather than being at rest there - to place material at rest there, it would have to be decelerated until the speed of light relative to it is c in both (or more accurately, all) directions.

The experiment (MGP, in this case), and the measurements are king: they give us numbers that tell us that R is greater than c and that c is greater than B, and we are duty bound as physicists to accept those facts rather than rejecting mathematics in a desperate attempt to cling to a dead theory. SR is in conflict with this experiment and is thereby invalidated, whereas LET is in full agreement with it. What should good, honest physicists do in such a situation? How should we judge these things? Do we judge the evidence and the argument, or do we ignore all that and just blindly follow a clergy? Should we judge by the qualifications of the person presenting the evidence (who in this case always claims to have none at all in order to avoid being followed on the basis of any kind of authority, although you really should wonder if he's hiding something when he has well-known mathematicians as uncles and a grandfather who hobnobbed with Gödel and Einstein), or should you go by the evidence itself? The correct answer is that you should always go by the evidence and argument and never be swayed by the identity of the person presenting it.
Title: Re: Can we measure the one way speed of light?
Post by: Bill S on 17/06/2018 11:30:22
I've not been following this thread, so this might well have been seen already.

https://medium.com/the-physics-arxiv-blog/the-one-way-speed-of-light-fcc5f05c5e44

If not, it might be of interest.
Title: Re: Can we measure the one way speed of light?
Post by: jeffreyH on 17/06/2018 14:57:51
How fast is the origin of a spacetime diagram moving?
Title: Re: Can we measure the one way speed of light?
Post by: PmbPhy on 17/06/2018 17:31:46
Quote from: David Cooper
Stop pretending that SR makes no such claims. SR claims that there is no aether,
That is an invalid assertion. SR only states that an ether plays no role in the speed of light. Einstein surely never meant to claim there was no either because in a speech he gave in Berlin on Jan.27th 1921 he wrote and presented lecture entitled Sidelights of relativity[/i] in which he said
Quote
More careful reflection teaches us, however, that the special theory of relativity does not compel us to deny ether. We may assume the existence of an ether; only we must give up ascribing a definite
state of motion to it, i.e. we must by abstraction take from it the last mechanical characteristic which Lorentz had still left it. We shall see later that this point of view, the conceivability of which I shall at once endeavour to make more intelligible by a somewhat halting comparison, is justified by the results of the general theory of relativity.
This lecture can be found online by just Googling the title.
Title: Re: Can we measure the one way speed of light?
Post by: David Cooper on 17/06/2018 20:10:17
I've not been following this thread, so this might well have been seen already.

https://medium.com/the-physics-arxiv-blog/the-one-way-speed-of-light-fcc5f05c5e44

If not, it might be of interest.

It's a typical example of people making assumptions and failing to understand why their experiment can't determine the one-way speed of light relative to the apparatus:-

"So Ahmed and co’s method is important. These guys create two identical pulses of light and send them in opposite directions along the same length. If there is any difference in the speed of these pulses, that ought to be detectable by photodiodes at each end of the experiment."

Any difference should not be detectable unless you synchronise your clocks on the basis that the apparatus is moving through space - the answers you get depend entirely on that synchronisation, and the standard way to synchronise them necessarily does so in such a way that builds in an assumption that the apparatus is at rest, so it's no surprise when the answers come back to "confirm" that.
Title: Re: Can we measure the one way speed of light?
Post by: David Cooper on 17/06/2018 20:21:10
Quote from: David Cooper
Stop pretending that SR makes no such claims. SR claims that there is no aether,
That is an invalid assertion.

That is possible, depending on which of Einstein's claims you consider to be part of SR and which you exclude from it because they aren't in particular papers. You yourself know that if you try to lay down the law about what SR is (on the basis of Einstein's papers) to the clergy on a certain physics forum that you can be banned for telling them what Einstein actually said SR is.

Quote
SR only states that an ether plays no role in the speed of light.

Isn't that bad enough? It's the aether (fabric of space) that imposes the speed limit on light. To deny that it has this role is to fly in the face of its crucial rule in actual relativity; a role which is exposed starkly by MGP.

What are your answers to my questions Q1 to Q5 (and to Q0: is R>B)? Do you have the courage to provide them? (I can't blame you if you don't want to say though, because you're more open to abuse by the clergy than the others here due to your position as an actual physicist, so you have a cast-iron excuse for staying silent.)
Title: Re: Can we measure the one way speed of light?
Post by: jeffreyH on 17/06/2018 20:24:19
How fast is the origin of a spacetime diagram moving?

I suppose no one has an answer to this.
Title: Re: Can we measure the one way speed of light?
Post by: David Cooper on 17/06/2018 20:39:04
How fast is the origin of a spacetime diagram moving?

I suppose no one has an answer to this.

It's always "at rest" in the chosen frame. The thing it's representing (if it maps to something real) may or may not be at rest in the actual universe.
Title: Re: Can we measure the one way speed of light?
Post by: jeffreyH on 17/06/2018 21:23:24
How fast is the origin of a spacetime diagram moving?

I suppose no one has an answer to this.

It's always "at rest" in the chosen frame. The thing it's representing (if it maps to something real) may or may not be at rest in the actual universe.

So define one spacetime diagram for the inertial frame and another for a frame in relative motion and see how it relates light between the two.
Title: Re: Can we measure the one way speed of light?
Post by: David Cooper on 17/06/2018 21:36:16
So define one spacetime diagram for the inertial frame and another for a frame in relative motion and see how it relates light between the two.

It relates it by changing the asserted speed of light relative to the two such that if one of them happens to show reality properly, the other shows a warped version where the asserted speed of light relative to the content is wrong (though you can never measure which is right or wrong from inside the universe).

Every time you change the frame of reference, you are producing a new theory of what the reality might be. (And if you use a non-inertial frame, you are mixing incompatible theories together in an unholy mess of contradiction.)
Title: Re: Can we measure the one way speed of light?
Post by: David Cooper on 18/06/2018 21:44:27
There's still a complete lack of people who are prepared to answer the simple questions I asked. Perhaps it's so clear that the answers have to be "yes" that they don't feel the need to do so. Assuming that this is the case, allow me to spell out the consequences of R>B in another way.

If R>B, there must be some parts of the ring where the speed of light relative to the material of the sectors it's passing through is greater in one direction than the opposite direction. If we now draw a tangent to the ring passing through such a point (or a straight line passing through that point along the direction in which that material is actually moving - the whole ring could be moving along, so this line isn't guaranteed to be a tangent to the ring), we can imagine an object Z moving along that line, temporarily co-moving with the material of the sector in question. The speed of light relative to Object Z in one direction along that line must also be greater than it is in the opposite direction - if this was not the case, we would have to have light moving at different speeds in the same direction on the same path, perhaps with one lot of light being visible to the ring material and magically hidden from Z, while the other lot would be visible to Z and magically hidden from the ring material, but we shouldn't be looking for magical theories like that.

If you want to deny that the speed of light relative to Z is different in these opposite directions, then you are forced to deny that the chosen point on the ring is in that situation too, which would mean that we've chosen the wrong point on the ring, but we can go through every point of the ring in turn and apply the same process to each of them in turn, and at least half of them must qualify as locations where the speed of light is different in opposite directions relative to the local ring material. If we deny that the speed of light relative to Z is different in opposite directions along the line in every case, we are left with no points on the ring where the speed of light can be different in opposite directions relative to the material of the sector it's passing through, but the result of that is that we must bury our heads in the sand and deny the result of the MGP (and the Sagnac) experiment. Serious physicists should not be doing this - they are duty bound to accept the results of these experiments and the consequences.

What is most interesting about all this though is that we have something directly equivalent to a religious belief in science - it is a case where programming (or systematic brainwashing by people who actively teach that LET is wrong) overrides people's ability to see even the most obvious contradictions. If you want to assert that the speed of light relative to Z in all directions must be c in every case, you are actually changing the frame of reference to do so, and in doing so you are changing the asserted speed of light relative to each object in order to pretend that it is equal to c in all directions. Whenever you do that though, you effectively assert that it is no longer c in all directions relative to some other objects which aren't co-moving with z, although you are banned from recognising that you are making this hidden assertion. You have been taught to keep changing the speed of light relative to an object until it's c in all directions (without understanding that that's what you're doing when you change frame), and then when you find other objects that don't conform to that because they're moving through the current frame of reference, you play the same game again with those - you use as many different frames of reference as necessary to assert that the speed of light relative to all objects is c in all directions, but these different frames of reference simply aren't compatible with each other - they are rival theories which each assert a different speed of light relative to the speed of light of other frames. If you try to apply two or more of them at once, this equates to an unspoken assertion that light moves along the same paths at different speeds (which again you are banned from recognising).

It may be that my analysis of MGP is your last hope of being deprogrammed of the SR mind virus that has blocked your ability to recognise contradictions. Here we have a clear case where we can measure the track round the ring by putting down measuring sticks all the way round it and verifying that it's the same length in both directions. We can time the light circuits relative to X (who moves with sector zero at all times) from any perspective with all observers confirming that the red light does indeed pass the material of all 100 sectors more times than the blue light, and they are forced (assuming they are rational) to conclude that the red light passes that material at a higher average speed relative to it (while it's passing it). This means that there must be some sectors where at some points in time the speed of light passing through them is moving at greater than c relative to them in one direction and less than c in the opposite direction. As soon as we accept that this is the case, we should be able to see that there must be an absolute frame to support this difference in the speed of light relative to the material content of space because frame Z manifestly cannot be a true representation of reality if Z (which is at rest in frame Z) doesn't have light moving relative to it at c in all directions, and if you want to rule out that Z as having that property, you are forcing other Zs to have that property instead.
Title: Re: Can we measure the one way speed of light?
Post by: jeffreyH on 19/06/2018 11:39:51
Spacetime diagrams have only 1 spacial dimension. They may be useful to compare separation over time but it would be better to have axes ct, x and y so that we can have the paths at least evolve on a plane.
Title: Re: Can we measure the one way speed of light?
Post by: Le Repteux on 19/06/2018 14:57:37
This is exactly why this is the best science forum. Is everyone taking this in? This is a very unique opportunity to learn. Don't waste it. No tuition fees!
I'm late, but what an interesting question Jeffrey. This is indeed by far the best forum on relativity, and it is so because Jeffrey is challenging it even if he doesn't say so and because David is there to replace him showing us how it works.  I think that the one way speed leads to a dead end for relativity, and that one day or another, it will be admitted. Meanwhile, I can again observe my theory on resistance being tested right before my naked eyes. I still have a question for those who think the one way speed of light can be measured to be c all the time though; who knows if I won't be luckier than David?

What's the use to know that the one way speed of light is always c if we never have to use it in real life?
 
No need for that data in the GPS for instance, and it works. As Einstein might have said, that data is superfluous, so why keep it? I do have to use a constant speed for c when I make my simulations with light (http://lumiere.shost.ca/) though, but it is so only because the screen works as a medium where light can travel at c in any direction. These simulations are mathematically and logically right otherwise they wouldn't give the right numbers and they do, and they show very clearly that the one way speed of light is not the same in both directions for the light clock that moves across the screen. So what is easier? Let down the ether without which we could not make any simulation of reality, or let down the one way speed that we can use nowhere but in our minds? Can you feel my resistance? :0)
Title: Re: Can we measure the one way speed of light?
Post by: guest4091 on 19/06/2018 18:01:38
David C #83;

Quote
If Ann is really stationary, her account represents reality while Ben's account is a distorted view of events.
---------
Red is based on what?
In the 'train' example, both observe the extended event 'object falls from passenger hand to floor', but some of the details vary for each. One reality, two descriptions. You aren't allowing for the changing position of the object due to relative motion, which results in different trajectories for the object. The trajectories are images/reality in the mind.
-------
Quote
My experiment, which you are still failing to address, demonstrates that there is a difference in the actual speed of light relative to the material of the ring in opposite directions as it travels through the sectors.
------
My answer was:
A, moving with the ring, will meet the cw photon 1st, then the ccw photon 2nd.  A explains the time difference as due to the rotation, not a difference in light speed, i.e. different path lengths. These are shown with the aide of the mirror.
If light clocks were used in the H-K experiment, the same results would occur, due to rotation. Although an absolute frame with acceleration, it can be analyzed with SR.
-------
Quote
Q1: If we have an observer X co-moving with S0 throughout, does he see the pulse of red light pass him more often than the blue light? (The correct answer is yes, and it is also yes for all observers - they see the red light pulse passing him more often than the blue one.)
-----

Title: Re: Can we measure the one way speed of light?
Post by: guest4091 on 19/06/2018 18:27:13
David C. #83;
Quote
You know that LET produces the same accounts of events to document the same perceptions.
Given a choice, LET or SR, I would choose SR since it doesn't require an ether, and light as particles doesn't require a medium.
Title: Re: Can we measure the one way speed of light?
Post by: jeffreyH on 19/06/2018 20:08:00
This is exactly why this is the best science forum. Is everyone taking this in? This is a very unique opportunity to learn. Don't waste it. No tuition fees!
I'm late, but what an interesting question Jeffrey. This is indeed by far the best forum on relativity, and it is so because Jeffrey is challenging it even if he doesn't say so and because David is there to replace him showing us how it works.  I think that the one way speed leads to a dead end for relativity, and that one day or another, it will be admitted. Meanwhile, I can again observe my theory on resistance being tested right before my naked eyes. I still have a question for those who think the one way speed of light can be measured to be c all the time though; who knows if I won't be luckier than David?

Hold on cowboy. Don't go putting words into my mouth or motives into my mind. Read what I actually post, not what you think I post
Title: Re: Can we measure the one way speed of light?
Post by: alancalverd on 19/06/2018 20:42:30
Here's another experimental idea. Consider the Doppler shift of received frequency

Δf = 2vf/(c - v)

So you can set up a laser, microwave transmitter, or radionuclide gamma source to provide a one-directional electromagnetic wave, and measure the Doppler shift between a fixed and a moving (e.g.oscillating) receiver. Now rotate the principal axis of the experiment around the receiver station, say from E-W to W-E,  and convince yourself that f and Δf remain the same, so c must be independent of direction.

Note that f varies between up and down - the Pound-Rebka experiment. The explanation depends on the constancy of c!
Title: Re: Can we measure the one way speed of light?
Post by: David Cooper on 19/06/2018 22:47:01
Quote
If Ann is really stationary, her account represents reality while Ben's account is a distorted view of events.
---------
Red is based on what?

If Ann is really stationary relative to the fabric of space.

Quote
In the 'train' example, both observe the extended event 'object falls from passenger hand to floor', but some of the details vary for each. One reality, two descriptions. You aren't allowing for the changing position of the object due to relative motion, which results in different trajectories for the object. The trajectories are images/reality in the mind.

Again there are multiple descriptions (theories) of this, but only one of them can be true - the rest are distorted views of the event.

Quote
Quote
My experiment, which you are still failing to address, demonstrates that there is a difference in the actual speed of light relative to the material of the ring in opposite directions as it travels through the sectors.
------
My answer was:
A, moving with the ring, will meet the cw photon 1st, then the ccw photon 2nd.  A explains the time difference as due to the rotation, not a difference in light speed, i.e. different path lengths. These are shown with the aide of the mirror.
If light clocks were used in the H-K experiment, the same results would occur, due to rotation. Although an absolute frame with acceleration, it can be analyzed with SR.

Why won't you answer the key question? Is R>B? (R is an average speed which you can build up by adding together the speed of the red light through each sector relative to that sector, while B is the equivalent for the blue light.) I've already helped you by telling you that the answer is yes, but I want to hear you answer it yourself. If R=B, observer X (equivalent to your A, but I don't want to use that name as it comes from a different experiment with a mirror in it where the light does something very different) would have both lots of light return to him simultaneously every time it passes him, but we know from the MGP and sagnac experiments that that doesn't happen.

If you insist that R=B, that is incompatible with the results of those experiments. If you accept that R>B though, you are necessarily accepting that the speed of light is higher relative to some of the material of the ring as it passes it in one direction than it is in the opposite direction, and by extension that there is an absolute frame. Where do you put your loyalties: with the universe or with a clergy? Galileo showed the clerics the moons of Jupiter through his telescope and they denied that they were there. Do you want history to look on you as being just like them?

Quote
Q1: If we have an observer X co-moving with S0 throughout, does he see the pulse of red light pass him more often than the blue light? (The correct answer is yes, and it is also yes for all observers - they see the red light pulse passing him more often than the blue one.)
-----

You've quoted one of my questions, but it isn't clear that you've answered it. Is your answer "yes"? You might score a point for being the first person other than me to dare to answer any of the questions.

Quote
Given a choice, LET or SR, I would choose SR since it doesn't require an ether, and light as particles doesn't require a medium.

Without a medium, there is no travel - you have nothing to support the properties of distance and direction, and nothing to impose a speed limit on light either. Most importantly though for this discussion, SR cannot handle this issue of the one-way speed of light relative to some objects being higher in one direction than the other. We know absolutely from MGP and Sagnac that such objects must exist where the speed of light is definitively not c relative to them in all directions (and indeed where it is >c in some directions), which means that any frame of reference Z that asserts that the speed of light relative to object Z in all directions is c is manifestly not a valid frame in that it cannot be representing reality, but a mere distortion of it. My analysis of the MGP experiment shows that some points with this property (that the speed of light relative to them varies in different directions) must exist and that any frame of reference which portrays them with those objects at rest cannot be a valid frame.
Title: Re: Can we measure the one way speed of light?
Post by: David Cooper on 19/06/2018 22:58:51
Here's another experimental idea. Consider the Doppler shift of received frequency

Δf = 2vf/(c - v)

So you can set up a laser, microwave transmitter, or radionuclide gamma source to provide a one-directional electromagnetic wave, and measure the Doppler shift between a fixed and a moving (e.g.oscillating) receiver. Now rotate the principal axis of the experiment around the receiver station, say from E-W to W-E,  and convince yourself that f and Δf remain the same, so c must be independent of direction.

Note that f varies between up and down - the Pound-Rebka experiment. The explanation depends on the constancy of c!

This sounds like a description of the experiment with a turntable, a detector at the middle and a source at the edge. The detector is tuned to the exact frequency of the source, so it doesn't pick up the signal unless it is exactly that frequency. When the table isn't turning, it detects nothing. When the table's turning round at a particular rate, the movement of the source slows its mechanism down (all moving clocks and mechanisms of any kind run slow) and the detector picks up the signal. If you move the whole turntable along through space, the functionality of the turntable, detector and source all slow down because of their movement though space, but the source is now speeding up and slowing down as it orbits the detector, so its frequency varies. The detector detects the signal from the source the whole time though, not recognising any variation, and this happens because the Doppler effect masks all the variation.

LET predicts the behaviour of all such experiments in full.
Title: Re: Can we measure the one way speed of light?
Post by: alancalverd on 20/06/2018 07:30:06
Wholly different experiment, wholly different result.

In my experiment the source does not move during the measurement.The only moving part is the locally oscillating part of the receiver, and the measurement is of the Doppler shift between the fixed and moving parts of the receiver when the direction of the incoming beam is fixed.

Your "ring experiments" are also irrelevant. c is the speed of light in vacuo, not in a medium. cm is subject to all sorts of variations.
Title: Re: Can we measure the one way speed of light?
Post by: Le Repteux on 20/06/2018 13:26:20
Hold on cowboy. Don't go putting words into my mouth or motives into my mind. Read what I actually post, not what you think I post
:0)
Sorry sheriff, I was just kidding. Now let's get serious: you said you were going to answer the first message from David, and you didn't. The way your OP was built almost meant that you were turning agnostic, that's exciting. How about being the first scientific forum to promote LET?
Title: Re: Can we measure the one way speed of light?
Post by: alancalverd on 20/06/2018 14:08:32
Here's another example. When an electron and a positron annihilate, they produce two photons, moving in exactly opposite (though unpredictable) directions and each with energy 511 keV. It is no coincidence that mp c2 = mec2 = 511 keV,and of course the photons can only move at c.

It is "quite" easy (it only took my team 20 man-years) to measure the energy of the photons by several different means, none of which involves any spatial dimension. To nobody's surprise, the energy of the left-right photon is exactly the same as that of the right-left photon and conservation of momentum thus demands that c is independent of direction.
Title: Re: Can we measure the one way speed of light?
Post by: Le Repteux on 20/06/2018 14:31:41
To nobody's surprise, the energy of the left-right photon is exactly the same as that of the right-left photon and conservation of momentum thus demands that c is independent of direction.
The speed of light is also independent of the direction of the source with LET if the source is at rest with regard to the fabric of space, and if the source is moving through space, it is impossible to observe any difference in the speed of its light due to relativity effects, which are the same with SR since they actually are relativity effects.
Title: Re: Can we measure the one way speed of light?
Post by: Colin2B on 20/06/2018 15:27:37
How about being the first scientific forum to promote LET?
How about we don’t, otherwise this discussion might end up in New Theories to make it clear LET is an alternative theory.
As it is, there is interesting and worthwhile discussion here on the technicalities and methods of looking at light speed.
Title: Re: Can we measure the one way speed of light?
Post by: Le Repteux on 20/06/2018 16:36:46
As it is, there is interesting and worthwhile discussion here on the technicalities and methods of looking at light speed.
SR is impossible to simulate on a computer screen, and LET is, so LET is better at summarizing reality even if both theories give out the same numbers.
Title: Re: Can we measure the one way speed of light?
Post by: alancalverd on 20/06/2018 17:37:57
Can anyone enlighten me as to the meaning of LET?
Title: Re: Can we measure the one way speed of light?
Post by: alancalverd on 20/06/2018 17:41:33
the fabric of space

Please set down the physical characteristics of this wondrous fabric. Density? Elastic modulus? Viscosity? εFS and μFS?

If I can move relative to it, what is it fixed to?
Title: Re: Can we measure the one way speed of light?
Post by: alancalverd on 20/06/2018 17:47:50
Never mind fairy gossamer or whatever you are on about, I have simply given a couple of experimental proofs that c is independent of direction so the one-way speed of light in vacuo is the same as the two-way value.

Some smart-arse will say "but you haven't measured it". To which I reply if two identical cans of beans cost $1, one can costs 50c. Or does fairy gossamer LET give quantity discounts?
Title: Re: Can we measure the one way speed of light?
Post by: Le Repteux on 20/06/2018 18:21:07
the fabric of space

Please set down the physical characteristics of this wondrous fabric. Density? Elastic modulus? Viscosity? εFS and μFS?

If I can move relative to it, what is it fixed to?
(LET is the acronym for Lorentz Ether Theory.) Space has no other characteristic than to let light and matter travel in it. Matter is considered to be traveling freely through it, though I have a particular viewpoint on that question, an light is considered to propagate through it, so it is not free from it. When we see a diagram of light traveling transversally between the two mirrors of a light clock for instance, we see light traveling through space while the mirrors are moving with regard to it. If it was shown traveling directly between the mirrors, the  diagram would mean that they are at rest with regard to it.
Title: Re: Can we measure the one way speed of light?
Post by: guest4091 on 20/06/2018 18:30:54
Bill S.;

Quote
I've not been following this thread, so this might well have been seen already.

https://medium.com/the-physics-arxiv-blog/the-one-way-speed-of-light-fcc5f05c5e44

If not, it might be of interest.

The experiment cited is misleading.
Sending light both ways simultaneously is equivalent to the 2-way experiment.
As the drawing shows, the 2nd signal could be sent in the opposite direction, in a duplicate device. As long as the signals return simultaneously, the events c1 and c2 are perceived as simultaneous.
The 2nd postulate, "the speed of light is independent of its source', implies 'its independent of the speed of the source'. Assuming U is the absolute rest frame whose observer Uri made the drawing, Uri concludes, Alice will always measure light speed as c, and perceive herself at rest, regardless of speed.
The 2nd postulate is another equivalence principle.
'An observer moving in uniform motion (free of changes in speed and direction) is equivalent to being at rest'.
If that is true, then the U frame is redundant.
 [ Invalid Attachment ]
Title: Re: Can we measure the one way speed of light?
Post by: Le Repteux on 20/06/2018 18:34:45
To which I reply if two identical cans of beans cost $1, one can costs 50c.
:0)
Two identical particles also carry the same energy, but if we observe their light while one is moving with regard to us and the other not, we don't get the same energy. In other words, we can't get rid of a relativity problem with classical physics.
Title: Re: Can we measure the one way speed of light?
Post by: guest4091 on 20/06/2018 18:35:37
David C;
Q1: If we have an observer X co-moving with S0 throughout, does he see the pulse of red light pass him more often than the blue light? (The correct answer is yes, and it is also yes for all observers - they see the red light pulse passing him more often than the blue one.)
-----
Wanted to make a simple drawing, since they show multiple relations easier than a page of text.
The drawing shows the ccw photon needing more time than the cw photon to complete 1 revolution relative to A. This is still a case of closing speed, since A is moving at .2 and the photon is moving at 1.0 in space. The average light speed would still be 1.0
If confused, imagine the flat surface wrapped around a tube with the 2 A-lines joined with the times aligned..
If m(1-v) = n(1+v), then v = (m-n)/(m+n). In this case v=1/5 = .2. The pattern repeats after 3 short and 2 long cycles.
 [ Invalid Attachment ]
Title: Re: Can we measure the one way speed of light?
Post by: David Cooper on 20/06/2018 20:07:39
Wholly different experiment, wholly different result.

In my experiment the source does not move during the measurement.The only moving part is the locally oscillating part of the receiver, and the measurement is of the Doppler shift between the fixed and moving parts of the receiver when the direction of the incoming beam is fixed.

If you move the receiver instead of the source, you speed up or slow down the functionality of the receiver in such a way as to maintain the perceived frequency again - this change in the speed of functionality and the Doppler effect always cancel out the differences that you imagine the experiment will detect if you don't bother to do the maths correctly to see that no effect should be detectable.

Quote
Your "ring experiments" are also irrelevant. c is the speed of light in vacuo, not in a medium. cm is subject to all sorts of variations.

If you read my experiment description carefully, it's a vacuum in a mirror-lined tube, although it works fine in a fibre-optic cable too because the delays from the medium are tiny.
Title: Re: Can we measure the one way speed of light?
Post by: David Cooper on 20/06/2018 20:09:39
Hold on cowboy. Don't go putting words into my mouth or motives into my mind. Read what I actually post, not what you think I post
:0)
Sorry sheriff, I was just kidding. Now let's get serious: you said you were going to answer the first message from David, and you didn't. The way your OP was built almost meant that you were turning agnostic, that's exciting. How about being the first scientific forum to promote LET?

That's asking a lot. This thread isn't about promoting LET either, but about discussing the evidence and looking at the point where SR fails to account for the facts.
Title: Re: Can we measure the one way speed of light?
Post by: David Cooper on 20/06/2018 20:12:58
Here's another example. When an electron and a positron annihilate, they produce two photons, moving in exactly opposite (though unpredictable) directions and each with energy 511 keV. It is no coincidence that mp c2 = mec2 = 511 keV,and of course the photons can only move at c.

How do you detect the energy of a photon? if your detector is moving away from the light, it records a lesser energy for it, so you're actually biasing your experiment by putting your detector at rest in a frame of reference in which your electron and positron system is also at rest.

Every time, your experiments contain a hidden bias that you've failed to recognise.
Title: Re: Can we measure the one way speed of light?
Post by: David Cooper on 20/06/2018 20:19:39
SR is impossible to simulate on a computer screen, and LET is, so LET is better at summarizing reality even if both theories give out the same numbers.

It's more complicated than that, but this thread is not here to promote LET. I only mention LET in it to show that a theory with variable speed of light relative to objects predicts the same results of experiments as SR - whenever someone claims that a difference in the one-way speed of light relative to an object would be detectable by an named experiment, I have to respond to that to explain why the experiment would not be capable of detecting any such thing - the person proposing it always has a naive understanding of what the experiment can do.
Title: Re: Can we measure the one way speed of light?
Post by: David Cooper on 20/06/2018 20:23:21
Never mind fairy gossamer or whatever you are on about, I have simply given a couple of experimental proofs that c is independent of direction so the one-way speed of light in vacuo is the same as the two-way value.

Some smart-arse will say "but you haven't measured it". To which I reply if two identical cans of beans cost $1, one can costs 50c. Or does fairy gossamer LET give quantity discounts?

You haven't produced any experimental proofs that the one-way speed of light in vacuo is the same as the two-way value. All you have done is present experiments which are incapable of determining anything relevant to that at all. In every single case, you build an assumption into it and then read that assumption back out at the end.
Title: Re: Can we measure the one way speed of light?
Post by: David Cooper on 20/06/2018 20:43:48
Q1: If we have an observer X co-moving with S0 throughout, does he see the pulse of red light pass him more often than the blue light? (The correct answer is yes, and it is also yes for all observers - they see the red light pulse passing him more often than the blue one.)

Again you've quoted the question, and again you haven't answered it. What you have done comes closer to addressing Q3:-

"Q3: Given that the red light returns faster than the blue light each time, should observer X conclude that the red light has passed through the sectors of the ring at a higher average speed relative to them (while it's passing through them) than the blue light? (The correct answer is yes: he should respect mathematics and conclude that R>B.)"

But have you shed any light on that? No. Remember what I said in reply #79:-

"[Note my careful wording: "average speed of light relative to each of the hundred sectors while it's passing through it" - if you ignore the "while it's passing though it" part, you will get a different result which would mask the truth of what's going on. My careful wording is there to force you only to consider the speed of light relative to any sector that it is passing through at the time while it's passing through it and not when the light is elsewhere in the circuit. It is only when you respect this wording that you force the truth out from the experiment.]"

You haven't done the maths correctly because you've ignored the specifications. To answer Q3 correctly you need to add up a series of numbers. For the red light, you have to take the speed of that light relative to sector S0 while it's travelling through S0, then add the speed of that light relative to sector S1 while it's travelling through S1, then add the speed of that light relative to sector S2 while it's travelling through S2, and so on all the way up to sector S99. Then you repeat that task for the blue light. The two values that you get can then be divided by 100 to get the averages, the result for the red light being the value R and the result for the blue light being the value B. If you do this correctly (using any frame of reference you like), you will find that R>B.

There is a simpler way to do the job though, because we can simply use the distance round the ring (which is the same in both directions) and the clock timings made by X at S0, and again we get R>B.
Title: Re: Can we measure the one way speed of light?
Post by: Le Repteux on 20/06/2018 20:47:07
Here is the conclusion at the bottom of the Wikipedia page on the one way speed of light:
Quote from: Wiki
In Einstein's special theory of relativity, all inertial frames of reference are equivalent and there is no preferred frame. There are theories, such as Lorentz ether theory that are experimentally and mathematically equivalent to special relativity but have a preferred reference frame. In order for these theories to be compatible with experimental results the preferred frame must be undetectable. In other words, it is a preferred frame in principle only, in practice all inertial frames must be equivalent, as in special relativity.

To me, this conclusion simply means that simulations using the screen as a rest frame are perfectly compatible with SR if we do not consider it as a preferred frame, which is the case for such simulations. As for SR, the rest frame is only chosen by convenience, so SR people should be allowed to study them, but those who take a look at them don't, because they can immediately see that the light is not moving at the same speed in both directions. We are actually neglecting a powerful tool to study motion just because Einstein said it was simpler to let down ether. I suspect he said that just to look different. He worked at the patent office, so he knew how to convince referees that his idea was new.
Title: Re: Can we measure the one way speed of light?
Post by: David Cooper on 20/06/2018 21:10:42
Here is the conclusion at the bottom of the Wikipedia page on the one way speed of light:
Quote from: Wiki
In Einstein's special theory of relativity, all inertial frames of reference are equivalent and there is no preferred frame. There are theories, such as Lorentz ether theory that are experimentally and mathematically equivalent to special relativity but have a preferred reference frame. In order for these theories to be compatible with experimental results the preferred frame must be undetectable. In other words, it is a preferred frame in principle only, in practice all inertial frames must be equivalent, as in special relativity.

The problem comes when you want to lay down the law about reality, and with SR there is a claim made by most of its followers that there is no absolute frame in reality (rather than just in the theory). However, what I've shown here (with MGP) is that some frames cannot be representations of reality. I have shown that some objects must exist where the speed of light relative to them is >c in some directions and <c in the opposite directions, and as soon as we have objects with this property, any frames of reference in which those objects are at rest are necessarily false frames. We can't tell which frames are the false frames, but we know that they exist and make incorrect assertions about the speed of light relative to those objects. SR is incapable of accounting for such false frames, labelling all frames as equally valid.
Title: Re: Can we measure the one way speed of light?
Post by: Le Repteux on 20/06/2018 21:14:14
This thread isn't about promoting LET either, but about discussing the evidence and looking at the point where SR fails to account for the facts.
The OP is from Jeffry, and it is intriguing, as if he wasn't sure SR was right and as if he wanted us to convince others. Why such a delicate question from a moderator if he is sure SR is right?
Title: Re: Can we measure the one way speed of light?
Post by: Le Repteux on 20/06/2018 21:17:54
SR is incapable of accounting for such false frames, labeling all frames as equally valid.
Such a mistake is impossible to make with simulations, so if people would only try to understand them, they would realize that SR is wrong.
Title: Re: Can we measure the one way speed of light?
Post by: Colin2B on 21/06/2018 08:45:57
This thread isn't about promoting LET either, but about discussing the evidence and looking at the point where SR fails to account for the facts.
The OP is from Jeffry, and it is intriguing, as if he wasn't sure SR was right and as if he wanted us to convince others. Why such a delicate question from a moderator if he is sure SR is right?
Looking at the OP it is a very straightforward question for which members have offered answers leading to some interesting discussion and it would be good to keep this thread on topic.
David, Jeff, Pete and I have discussed LET before and I recognise the common ground and respect David’s intellect, but as he says this is not about promoting LET, that should be done elsewhere as an new/alternative theory.
Title: Re: Can we measure the one way speed of light?
Post by: alancalverd on 21/06/2018 09:45:58

You haven't produced any experimental proofs that the one-way speed of light in vacuo is the same as the two-way value. All you have done is present experiments which are incapable of determining anything relevant to that at all. In every single case, you build an assumption into it and then read that assumption back out at the end.

On the contrary. I have referenced Maxwell's ab initio calculation of the speed of an electromagnetic wave which, being independent of any medium, is independent of direction, and an everyday experiment that shows the energy of a pair-annihilation photon mec02 is independent of direction. If cA→B = cB→A then any measurement of c in vacuo is equivalent to its one-way speed.

A philosopher might argue that I can't see a tree. Being a really clever smartarse he would say that I am receiving the photons emitted by the tree a few nanoseconds ago. Bishop Berkeley dismissed such pointless hairsplitting in 1710. I think physics has moved on since then.

If you want  cA→B > cB→A you will have to answer the questions about the physical properties of the "fabric of space", show us its universal axes of anisotropy, and explain why Maxwell's equations are correct.
Title: Re: Can we measure the one way speed of light?
Post by: Le Repteux on 21/06/2018 12:43:43
David, Jeff, Pete and I have discussed LET before and I recognize the common ground and respect David’s intellect, but as he says this is not about promoting LET, that should be done elsewhere as an new/alternative theory.
It is not LET that I'm promoting, its my simulations (http://lumiere.shost.ca/). If they are built correctly, simulations work like math or time/space diagrams: they are a means for representing reality. It's David that showed me how to build them in Java Script language, which is quite close to HTML, which is quite easy to learn, and I knew a bit of it, so it took me only about two weeks to be on my own, and I'm far from being a genius, so I think almost anybody can check them out if he wants, but nobody did yet. It's abnormal, such a nice tool and nobody cares, because everybody think  that relativity is right all the way, so they also think that anything that is presented as a critic must necessarily be wrong. What's wrong about trying to represent reality on a computer screen? Start this one (http://lumiere.shost.ca/Twins%20paradox/Twins%20paradox.html), and tell me if the photon in the moving light clock takes the same time going right than going left between the mirrors.
Title: Re: Can we measure the one way speed of light?
Post by: Colin2B on 21/06/2018 13:33:47
It is not LET that I'm promoting, its my simulations
Happy to take a look, left you a message in the New Theories section, but would still like to keep this thread on the specific OP question.
Title: Re: Can we measure the one way speed of light?
Post by: guest4091 on 21/06/2018 16:49:17
David .C;
In #114, my quote;
"The drawing shows the ccw photon needing more time than the cw photon to complete 1 revolution relative to A."
"The pattern repeats after 3 short and 2 long cycles."

If you can't translate that into 'there are more short cycles than long cycles, you apparentaly have a comprehension ptoblem.

your quote
Quote
If you move the whole turntable along through space, the functionality of the turntable, detector and source all slow down because of their movement though space, but the source is now speeding up and slowing down as it orbits the detector, so its frequency varies.

1. Where does the source get the extra energy to speed up and slow down?
2. There is no relative motion between source and detector, if their speparation is a constant radius.

You continuosly use the term "actual reality", but never show how to find it.
Title: Re: Can we measure the one way speed of light?
Post by: David Cooper on 21/06/2018 17:44:20
On the contrary. I have referenced Maxwell's ab initio calculation of the speed of an electromagnetic wave which, being independent of any medium, is independent of direction, and an everyday experiment that shows the energy of a pair-annihilation photon mec02 is independent of direction. If cA→B = cB→A then any measurement of c in vacuo is equivalent to its one-way speed.

It is clearly not independent of direction. If you're moving towards a photon, you measure it as carrying higher energy than if you're moving in the opposite direction when it hits your detector. You cannot measure its speed, but merely its apparent frequency. If your detector is co-moving with the experiment, you will always get the same measurements regardless of how fast the system is moving through space and of how different the one-way speeds of light are across it in opposite directions.

Quote
If you want  cA→B > cB→A you will have to answer the questions about the physical properties of the "fabric of space", show us its universal axes of anisotropy, and explain why Maxwell's equations are correct.

The only required property of the space fabric for this is that it sets a limit for the speed of light through it. Maxwell's equations (or the way they're being applied) are a simplification which depends on relativity for producing the same answers regardless of speed through space. If you want to remove that simplification, you need to feed in alternative distances (wherever distance values are involved) based on what they would be for any other speed than zero through the space fabric. For example, if the system is moving at 0.866c to the right and a force carrier has to move from left to right, it will have to move than seven times as far through space to complete the trip as it would if the system was stationary, while a force carrier moving the opposite way would move a little more than half as far as if the system was stationary. The directions in which force carriers travel would also be adjusted in accordance with the headlights effect, and the separation between the point the force carrier's leaving and the point where it lands would be halved. The reason you don't normally have to bother complicating the calculations in this way is that you get good answers without going to so much trouble - you can simply rely on relativity masking all the differences from you, so your naive, simplistic approach works every time, but you have failed to understand how it works and how it hides the underlying reality from you, to the point that you misuse this calculation shortcut to assert that the one-way speed of light relative to the system is always equal to c. It displays a severe lack of understanding of the mathematics of relativity on your part, but you're not alone in that - education is to blame for this failure because only half the story of relativity's being taught in universities (and the more important half of the story is the one that's being missed out).
Title: Re: Can we measure the one way speed of light?
Post by: David Cooper on 21/06/2018 18:04:21
David .C;
In #114, my quote;
"The drawing shows the ccw photon needing more time than the cw photon to complete 1 revolution relative to A."
"The pattern repeats after 3 short and 2 long cycles."

If you can't translate that into 'there are more short cycles than long cycles, you apparentaly have a comprehension ptoblem.

I'm just trying to get you to commit to a clear "yes" or "no" answer. If I take your answer to Q1 as "yes", then there are five more questions there waiting for answers (Q2 to Q5, and Q0 [named retrospectively, but it was the original question: "Q0. Is R>B?"]).

Quote
1. Where does the source get the extra energy to speed up and slow down?

If we have a turntable rotating and moving along through space, the centre of the turntable is moving along through space at a constant speed while a point on the edge of the turntable is speeding up and slowing down (relative to the space fabric). For every part of the turntable that is slowing down, another part is speeding up, so the energy involved is constant. The speed that a rotating source/detector (on the edge of the turntable) moves at through space changes throughout a revolution, and as it changes, the functionality of that device speeds up and slows down - faster movement through space leads to slower functionality (due to increased cycle distances within the component).

Quote
2. There is no relative motion between source and detector, if their speparation is a constant radius.

The radius is not constant due to length contraction - a moving disc becomes elliptical. In addition to that, the communication distances through space vary because a signal from the trailing edge to the centre takes longer to reach the centre than a signal from the leading edge.

Quote
You continuosly use the term "actual reality", but never show how to find it.

Relativity hides it from us, so all we can see is an infinite range of possible realities which are incompatible with each other as they contradict each other by asserting different speeds of light relative to specific objects. The beautiful story of how relativity works is clearly something you have yet to see, because if you understood the maths of it you wouldn't have needed to ask those questions. It is the most extraordinary phenomenon, and yet it makes full sense once you've thought it all through and seen how the one-way speed of light relative to you is always masked. Our inability to pin down the absolute frame is a direct consequence of the speed of light being the speed limit governing all components of any experiment we build - we could only break though that measurement barrier by accessing something that moves through space faster than c and using it to send information, but we don't have that.
Title: Re: Can we measure the one way speed of light?
Post by: jeffreyH on 22/06/2018 11:42:00
The layman already has to navigate a confusing sea of mathematical formulas in trying to understand science. Asking the right question is important to bring to the fore concepts that are difficult to understand. Threads like these are very important since posters have to think before they answer and a real debate can happen. We have members with a wealth of knowledge on both sides of any argument. The points raised and answered offer a path through a difficult subject and allow the enthusiast to pursue those concepts through study. That can never be bad. What is bad is pseudoscientific propaganda and we have to guard against that. As I said, the right questions to ask bring out the best scientific debate. In order to ask those questions requires an understanding of the topic.
Title: Re: Can we measure the one way speed of light?
Post by: alancalverd on 22/06/2018 13:20:42
Maxwell's equations (or the way they're being applied) are a simplification which depends on relativity for producing the same answers regardless of speed through space. I
As I have said before, they are not a simplification and have nothing to do with relativity. They are the means by which we explain the observed selfpropagation of  electromagnetic radiation, from well-characterised phenomena that do not involve the movement of anything in any particular direction.

Relativity derives from the Maxwellian fact that c is not a vector.
Title: Re: Can we measure the one way speed of light?
Post by: Le Repteux on 22/06/2018 14:52:54
Relativity derives from the Maxwellian fact that c is not a vector.
Half of Einstein's 1905 first paper on special relativity, "On the Electrodynamics of Moving Bodies," explains how to transform Maxwell's equations.(exert from wiki page on classical electromagnetism and special relativity) (https://en.wikipedia.org/wiki/Classical_electromagnetism_and_special_relativity)

Vector or not, these equations can be transformed into more complicated relativity equations, so they are de facto simpler.
Title: Re: Can we measure the one way speed of light?
Post by: David Cooper on 22/06/2018 19:49:50
As I have said before, they are not a simplification and have nothing to do with relativity. They are the means by which we explain the observed selfpropagation of  electromagnetic radiation, from well-characterised phenomena that do not involve the movement of anything in any particular direction.

Relativity derives from the Maxwellian fact that c is not a vector.

Can you show me how you're crunching numbers with this to produce a one-way speed of light value and demonstrate that you aren't smuggling in any distance measurements.
Title: Re: Can we measure the one way speed of light?
Post by: guest4091 on 23/06/2018 17:29:52
David D. #120;

In making an improved drawing, I realized the previous one was in error. It showed the ring in motion when it should have been at rest relative to U.
Fig.1 without the A path (red) simply shows ccw photon path (blue) and cw path (green) leaving the origin simultaneously and returning simultaneously within the ring.
It forms a tube with light spiraling in circles along the time line for U.

With a device A attached to the ring,  the ring rotates ccw at .2c. A completes 90 deg at Ut= 7.7 and continues 90 deg on the far side of the tube (dashed red). U sees no change in the behavior of light.

Fig.2 displays the tube unrolled and flattened to show when the light intersects the A path. The results are the same as before, with A crossing the green path 3x and the blue path2x. This pattern then repeats.

If A records two speeds for light, .80 and 1.20, what is the standard?

 [ Invalid Attachment ]
Title: Re: Can we measure the one way speed of light?
Post by: yor_on on 23/06/2018 17:54:49
Yoron#62;
Thanks for sharing your thoughts.
I do think if many words do not impart understanding, your effort is wasted. I remind myself to keep it simple and clear for the reader, but am not always successful. When reviewing papers I've written in the past, if they seem faulty, they go in the trash. I'm my worst critic, so it's not a question of 'how good you think you are', but how well you convey understanding to others. I'll work to improve.

Thank you Phyti. You're cool with me.
And, in my eyes, you're doing a good job of simplifying it, as I read further on.

I know how hard that is.
It takes time, and somehow it never finish :)
Title: Re: Can we measure the one way speed of light?
Post by: David Cooper on 23/06/2018 21:03:32
In making an improved drawing, I realized the previous one was in error. It showed the ring in motion when it should have been at rest relative to U.

That's confusing. If the ring's in motion (rotating) and you want to show it at rest, you'd either have to show light moving at different angles up the diagram (meaning different amounts of tilt away from the vertical) or else have it show a result incompatible with experiments. The way you're doing it now though, you aren't putting the ring at rest relative to U because A is supposed to be rotating with the material of the ring. It's also confusing when you keep using A though when A came from a scenario with a mirror in it. It would be clearer if you either switched to using my names for observers or set out an alternative set of names for my experiment if you aren't happy with X (moving round with the ring material), Y (next to the ring but not going round with it), and S0 to S99 (the names of 100 sectors of the ring, with X always being with S0). The light paths should also be coloured red (moving clockwise) and blue (going anticlockwise). Once you start rotating your ring, the origin of frame U is where I put Y, while my X is equivalent to your A, moving round with S0.

Quote
If A records two speeds for light, .80 and 1.20, what is the standard?

What do you mean by "standard"? Observer A records that the average speed of the red light (which you've coloured green) is 1.2c relative to the material of the ring as it passes it (sector by sector), and that the average speed of the blue light (shown as blue) is 0.8c relative to the material of the ring as it passes it (sector by sector). If you want to average those to try to hide the difference and to go on to assert that the light must all have passed each sector relative to the material of that sector at c, then that would also mean that if a car passes you at 100mph in one direction and then passes you at 50mph relative to you the other way, it actually passed you at 75mph relative to you both times. Both of those are departures from mathematics. We clearly have two different average speeds for light relative to the same material, namely R and B, and R is greater than B.

Note that we could change our frame of reference to make the speed of the red light the same as the speed of the blue light relative to observer A at one point in A's journey round the ring, but that would need to be countered by an amplified difference in the speed of light relative to A in opposite directions when it's round the other side of the circuit - there always have to be places along the way when A is exposed to light travelling at different speeds in opposite directions relative to it. It is mathematically impossible for the red and blue light to pass all the material it passes at c relative to that material (while it's local to that material) and to get the end result R>B.
Title: Re: Can we measure the one way speed of light?
Post by: guest4091 on 25/06/2018 16:18:23
David D. #;

Quote
Q1: If we have an observer X co-moving with S0 throughout, does he see the pulse of red light pass him more often than the blue light? (The correct answer is yes, and it is also yes for all observers - they see the red light pulse passing him more often than the blue one.)
------
the drawings show this
------
Quote
Q2: Does observer X measure the ring as having the same length in both directions from S0 round the ring and back to S0? (The correct answer is yes, and it is also yes for all other observers.)
----
A is not moving relative to the ring, and would average the times of returning light.
----
Quote
Q3: Given that the red light returns faster than the blue light each time, should observer X conclude that the red light has passed through the sectors of the ring at a higher average speed relative to them (while it's passing through them) than the blue light? (The correct answer is yes: he should respect mathematics and conclude that R>B.)
------
Notice your word association, 'red light' with 'faster'.
In mine, closing speed of (green and A) 1.2, is greater than that for (blue and A) .8.
Light speed is 1.
-------
Quote
Q4: If we have an observer Y next to the rotating ring such that X is with him initially, but then X goes round with the ring next to sector S0 and eventually returns to Y at a moment when the pulses of red and blue light happen to arrive there simultaneously, should observer Y also conclude that the red light has passed through the sectors of the ring at a higher average speed relative to them (while it's passing through them) than the blue light? (The correct answer is yes: he too should respect mathematics and conclude that R>B.)
------
U at rest relative to the center of the ring and not rotating, sees green and blue make synchronized revolutions, independently of ring rotation. Light speed is independent of its source. Light speed is 1.
------
Quote
Q5: Do all observers have a perception which should lead them to conclude that R>B? (The correct answer is yes - all possible observers recognise that the red light has travelled through the 100 sectors at a higher speed relative to them on average than the blue light, assuming that they respect mathematics.)
----
NO! The MGP experiment is not measuring the 1-way speed of light.
From the Wiki article: https://en.wikipedia.org/w/index.php?title=Michelson–Gale–Pearson_experiment&oldid=736473424

"The aim, as it was first proposed by Albert A. Michelson in 1904 and then executed in 1925, was to find out whether the rotation of the Earth has an effect on the propagation of light in the vicinity of the Earth." (also predicted in 1911 by Max van Laue, using SR).

"The outcome of the experiment was that the angular velocity of the Earth as measured by astronomy was confirmed to within measuring accuracy"

"Thus special relativity is the only theory which explains both experiments (hypothesis of complete aether drag and MGP). The experiment is consistent with relativity for the same reason as all other Sagnac type experiments. That is, rotation is absolute in special relativity, because there is no inertial frame of reference in which the whole device is at rest during the complete process of rotation, thus the light paths of the two rays are different in all of those frames..."
A related experiment:
The time dilation effects in the H-K experiment resulted from Earth rotation, not variation in light speed.

Light speed is measured relative to a medium (real or imagined), not to an object.
If light propagation is a physical phenomenon, and the rules of physics are the same in all inertial frames, then light speed must be constant in all inertial frames.
Title: Re: Can we measure the one way speed of light?
Post by: guest4091 on 25/06/2018 16:31:32
David D. #131;
Quote
If we have a turntable rotating and moving along through space, the centre of the turntable is moving along through space at a constant speed while a point on the edge of the turntable is speeding up and slowing down (relative to the space fabric). For every part of the turntable that is slowing down, another part is speeding up, so the energy involved is constant. The speed that a rotating source/detector (on the edge of the turntable) moves at through space changes throughout a revolution, and as it changes, the functionality of that device speeds up and slows down - faster movement through space leads to slower functionality (due to increased cycle distances within the component)
.
----
Then you are not in the inertial frame containing the device. The MGP experiment was done in a lab just as the MM experiment. Neither experiment was influenced by the translational motion of the earth in space at 30km/sec. relative to the sun.
The more refined laser ring gyros, developed from the interferometer experiments, are in common use today and work across a wide range of speeds. Reference:
https://en.wikipedia.org/w/index.php?title=Sagnac_effect&oldid=843575092
Title: Re: Can we measure the one way speed of light?
Post by: Le Repteux on 25/06/2018 16:39:30
If light propagation is a physical phenomenon, and the rules of physics are the same in all inertial frames, then light speed must be constant in all inertial frames.
Not true! The laws of physics can be the same without the one way light speed being constant, and that's precisely what the MM experiment shows. He took for granted that ether existed, so all his calculations and drawings are based on that principle.

Quote
Neither experiment was influenced by the translational motion of the earth in space at 30km/sec. relative to the sun.
I think it's precisely what David was telling you.
Title: Re: Can we measure the one way speed of light?
Post by: guest4091 on 25/06/2018 16:52:03
If light propagation is a physical phenomenon, and the rules of physics are the same in all inertial frames, then light speed must be constant in all inertial frames.
Not true! The laws of physics can be the same without the one way light speed being constant, and that's precisely what the MM experiment shows. He took for granted that ether existed, so all his calculations and drawings are based on that principle.
If 'he' refers to Einstein, read the 1905 paper again. The 2nd postulate for a constant c is part of the theory. He also stated the ether wasn't needed..
Title: Re: Can we measure the one way speed of light?
Post by: Le Repteux on 25/06/2018 16:57:41
It's Michelson that took for granted that ether existed, not Einstein, and for him, it meant that the speed of light was different going right than going left in the interferometer, which is precisely what his drawings and calculations show.
Title: Re: Can we measure the one way speed of light?
Post by: David Cooper on 25/06/2018 21:27:28
Quote
Q1: If we have an observer X co-moving with S0 throughout, does he see the pulse of red light pass him more often than the blue light? (The correct answer is yes, and it is also yes for all observers - they see the red light pulse passing him more often than the blue one.)
------
the drawings show this

So that's a yes.

Quote
Quote
Q2: Does observer X measure the ring as having the same length in both directions from S0 round the ring and back to S0? (The correct answer is yes, and it is also yes for all other observers.)
----
A is not moving relative to the ring, and would average the times of returning light.

Did you read the question carefully? It's asking you about a length (or rather two lengths of the same thing which are clearly equal because they are the same length measured in opposite directions). Let's say that A has very long arms and can reach across to any part of the ring with a ruler to measure the length of each sector of the ring - he can do this starting with S0, then S1, then S2, etc, or he can measure it in the opposite direction by starting with S0, then S99, then S98, etc. Do you imagine that he gets a different length measurement of the ring in these two directions? No, he does not. That is how he should measure the distance round the ring from his non-inertial frame of reference.

Quote
Quote
Q3: Given that the red light returns faster than the blue light each time, should observer X conclude that the red light has passed through the sectors of the ring at a higher average speed relative to them (while it's passing through them) than the blue light? (The correct answer is yes: he should respect mathematics and conclude that R>B.)
------
Notice your word association, 'red light' with 'faster'.
In mine, closing speed of (green and A) 1.2, is greater than that for (blue and A) .8.
Light speed is 1.

Where I said "faster", you should interpret that as a time measure rather than speed - to clear up the ambiguity, feel free to replace it with "sooner". As you say, light speed is 1 (as always), while the speed of light relative to the light is 1.2 for the red/green light and 0.8 for the blue (which you call the closing speed). The one-way speed of light is always c relative to the fabric of space, but relative to the material of the ring at any point where the light is adjacent to that material, the one-way speed of the light to the ring is 1.2c on average for the red light and 0.8c on average for the blue.

Quote
Quote
Q4: If we have an observer Y next to the rotating ring such that X is with him initially, but then X goes round with the ring next to sector S0 and eventually returns to Y at a moment when the pulses of red and blue light happen to arrive there simultaneously, should observer Y also conclude that the red light has passed through the sectors of the ring at a higher average speed relative to them (while it's passing through them) than the blue light? (The correct answer is yes: he too should respect mathematics and conclude that R>B.)
------
U at rest relative to the center of the ring and not rotating, sees green and blue make synchronized revolutions, independently of ring rotation. Light speed is independent of its source. Light speed is 1.

Again (like with Q2) you have not answered the question that was asked. The question is about how fast the light passed the material of the ring while adjacent to it. Observer Y has seen the ring do a complete rotation and has seen the red and blue light go round several times - he knows that the red light passed each sector one extra time compared with the blue light, so he calculates that the red light was moving at a higher speed relative to that material (while adjacent to it) than the blue light was.

Quote
Quote
Q5: Do all observers have a perception which should lead them to conclude that R>B? (The correct answer is yes - all possible observers recognise that the red light has travelled through the 100 sectors at a higher speed relative to them on average than the blue light, assuming that they respect mathematics.)
----
NO! The MGP experiment is not measuring the 1-way speed of light.
From the Wiki article: https://en.wikipedia.org/w/index.php?title=Michelson–Gale–Pearson_experiment&oldid=736473424

"The aim, as it was first proposed by Albert A. Michelson in 1904 and then executed in 1925, was to find out whether the rotation of the Earth has an effect on the propagation of light in the vicinity of the Earth." (also predicted in 1911 by Max van Laue, using SR).

You're playing a blatant game of avoidance here. All observers conclude that the red light passed the material of the ring at an average speed >c while adjacent to it. This has nothing to do with the aim of the MGP experiment (which I only brought into this to show that when you rotate a ring, the light doesn't take the same length of time to return to observer X in both directions).

Quote
"Thus special relativity is the only theory which explains both experiments (hypothesis of complete aether drag and MGP).

And I pointed out earlier that the text of that Wikipedia page contradicts itself here because it has already admitted that LET also explains both experiments. My thought experiment shows though that SR is failing to account for one thing that LET can account for, and that is the differences in the one-way speed of light relative to objects which manifestly must occur in order to produce the timings that we get from MGP and Sagnac.

Quote
The experiment is consistent with relativity for the same reason as all other Sagnac type experiments. That is, rotation is absolute in special relativity, because there is no inertial frame of reference in which the whole device is at rest during the complete process of rotation, thus the light paths of the two rays are different in all of those frames..."
A related experiment:
The time dilation effects in the H-K experiment resulted from Earth rotation, not variation in light speed.

The light paths can vary wildly for different frames, but in every single frame we have the same light passing the same material with the same average speed difference for the red and blue light as it passes that material.

Quote
Light speed is measured relative to a medium (real or imagined), not to an object.
If light propagation is a physical phenomenon, and the rules of physics are the same in all inertial frames, then light speed must be constant in all inertial frames.

Each frame asserts that the speed of light relative to it (the frame) is c in all directions. If an object is at rest in frame A and is moving in frame B, then frame A asserts that the one-way speed of light relative to that object is c in all directions, while frame B asserts that the one-way speed of light relative to that object is >c in some directions and <c in others. It is not possible for both frame A and frame B's accounts to be true - they are both theories as to the speed of light relative to that object and it cannot be the case that the speed of light relative to that object is BOTH c in all directions AND >c in some directions and <c in others. SR makes out though that all frames are equally valid and that there is no absolute frame, but that cannot be so. We have objects which categorically do not have the speed of light relative to them equal to c in all directions, and any frame that makes out that the speed of light relative to them is c in all direction is a fake frame, misrepresenting the speed of light relative to them. Worse, if one frame represents the truth about the speed of light relative to an object in all directions, ALL other frames are misrepresentations of reality. All frames of reference contradict each other fundamentally.
Title: Re: Can we measure the one way speed of light?
Post by: David Cooper on 25/06/2018 21:47:15
David D. #131;
Quote
If we have a turntable rotating and moving along through space, the centre of the turntable is moving along through space at a constant speed while a point on the edge of the turntable is speeding up and slowing down (relative to the space fabric). For every part of the turntable that is slowing down, another part is speeding up, so the energy involved is constant. The speed that a rotating source/detector (on the edge of the turntable) moves at through space changes throughout a revolution, and as it changes, the functionality of that device speeds up and slows down - faster movement through space leads to slower functionality (due to increased cycle distances within the component)
.
----
Then you are not in the inertial frame containing the device.

If I put the turntable in a rocket which is moving at a constant speed in a single direction through space, that system can be analysed using inertial frames. The centre of the turntable does not move relative to the rocket other than rotating. If the speed of the rocket is zero, the speed of any object attached to the edge of the rotating turntable is constant. If the speed of the rocket is >0, the speed through space of the object on the edge of the rotating turntable will vary. Any change in the speed of the object at the edge of the rotating turntable will lead to its functionality speeding up or slowing down, but this change will be masked 100% from a device at the centre of the turntable by the Doppler effect.

Quote
The MGP experiment was done in a lab just as the MM experiment. Neither experiment was influenced by the translational motion of the earth in space at 30km/sec. relative to the sun.
The more refined laser ring gyros, developed from the interferometer experiments, are in common use today and work across a wide range of speeds. Reference:
https://en.wikipedia.org/w/index.php?title=Sagnac_effect&oldid=843575092

Why are you connecting these two things at all? The turntable issue is quite separate from my thought experiment (which has been done as an actual experiment in the form of MGP and Sagnac). The turntable issue is about frequencies generated and perceived as they travel between centre and edge, and it was Alan that brought it up as an experiment that would supposedly show up any change in the one-way speed of light relative to the apparatus. It can't show up any such change though because the changes in speed of the component at the edge of the table has its functionality rate changed by that change in movement speed, precisely masking out the change (when you also factor in the Dopplar shifts, the length contraction of the turntable into an ellipse, and the additional change of speed of the object at the edge of the turntable relative to the centre caused by it moving more slowly forwards than backwards - it takes longer to complete one half of a rotation than the other).
Title: Re: Can we measure the one way speed of light?
Post by: guest4091 on 26/06/2018 20:17:34
David C.;
From a distance, A observes B moving at .4c on path x, and a light pulse from A moving at c, on the same path. A calculates A and the pulse, separating at .6c. The pulse reflects from an object on the path, and A calculates A and the pulse, converging at 1.4c.
B calculates the speed of the pulse as c=1, based on round trip transit time.
To calculate the speed of the pulse relative to himself, B would have to know his speed in space. He has no means of detecting this.

Neither A nor B observe anything moving at .6c or 1.4c.
Title: Re: Can we measure the one way speed of light?
Post by: David Cooper on 26/06/2018 21:37:38
David C.;
From a distance, A observes B moving at .4c on path x, and a light pulse from A moving at c, on the same path. A calculates A and the pulse, separating at .6c. The pulse reflects from an object on the path, and A calculates A and the pulse, converging at 1.4c.

That's confusing. Shouldn't it say that A calculates that B and the pulse are separating at .6c and that after the refliction, B and the pulse are converging at 1.4c?

Quote
B calculates the speed of the pulse as c=1, based on round trip transit time.
To calculate the speed of the pulse relative to himself, B would have to know his speed in space. He has no means of detecting this.

Neither A nor B observe anything moving at .6c or 1.4c.

A calculates that the light is moving at those speeds relative to B, and if B reckons itself to be stationary, B will calculate that light is moving at those speeds relative to A.
Title: Re: Can we measure the one way speed of light?
Post by: David Cooper on 26/06/2018 22:13:57
I'm now going to set out a similar thought experiment where the ring is eliminated so that practically all the action can take place on a straight line. Imagine a loop of fibre-optic cable (or a flexible tube with silvered interior and a vacuum for the light to travel through). The two ends go round pulleys on the end of a long rod, but most of the cable is spread out in two straight lines close together, the rod between them. The loop of cable is divided into 100 sectors just like the ring, and an observer X will travel round with sector S0. Observer Y stays by one of the pulleys. X starts at that same pulley and then travels round with S0, eventually returning to where Y is. Red light is sent one way through the cable from S0 and blue light is sent the opposite way. By the time X has moved from Y's pulley to the far pulley and back, the light has done several laps of the cable.

The advantage of doing this is that we can make clear statements about the speed of light relative to the material for almost 50% of the material of the course in one go instead of only talking about one point on the ring.

As before, the red light passes X once more than the blue light does by the time X has completed his journey away from Y and back. Again, the red light has passed through the material of the loop that's directly adjacent to it at a higher speed relative to it than the blue light did. We know that we have material here which has light moving relative to it at >c in one direction and <c in the other. If the material in the part of the loop running along over the rod doesn't have this property, then the material in the part of the loop running along under the rod must. Conversely, if the material in the part of the loop running along under the rod doesn't have this property, then the material in the part of the loop running along over the rod must. The third option is that the material in both parts of the loop, both over and under the rod, all has this property. Every frame of reference asserts one of those three possibilities. There is no frame of reference that asserts none of the three options. If the top material has this property, any frame of reference that asserts that it doesn't is automatically a misrepresentation of reality, but if such a frame making that assertion is not a misrepresentation of reality, then the bottom material has that property instead, and any frame asserting that it doesn't is wrong. It is a clear case where it is not possible for both those frames of reference to be correct representations of reality - one of them is necessarily a distortion of the truth.

Let's add two objects W and Z, one co-moving with the lower part of the material of the loop and the other moving with the upper part. We know too that at least one of them must have light moving relative to it at speeds >c in one direction relative to it and <c in the opposite direction, and the thought experiment proves that in any case with W and Z moving relative to each other this will still apply even if we remove the rest of the apparatus. If one frame of reference removes this property from W and another removes it from Z, it is mathematically certain that at least one of those frames is a misrepresentation of reality.

Frames of reference flatly contradict each other - they are not a set of equally correct representations of the underlying reality.
Title: Re: Can we measure the one way speed of light?
Post by: guest4091 on 28/06/2018 15:27:28
That's confusing. Shouldn't it say that A calculates that B and the pulse are separating at .6c and that after the refliction, B and the pulse are converging at 1.4c?
Yes it should, if I don't make mistakes!
Reciprocity requires that swapping A and B, yields the same results except B is moving in the opposite direction. The fact still remains that neither measure the speed of any object moving at the closing speeds.
Title: Re: Can we measure the one way speed of light?
Post by: guest4091 on 28/06/2018 15:31:40
David D. #;

If a rotating device made on Earth is working as designed, and put into a space probe and sent on a mission, the device continues to work as designed, despite moving at high speed relative to Earth. The motion of the device relative to any 'ether' is irrelevant, since measurements by and on the device are done within the device frame of reference. The motion of the Earth, sun, galaxy, etc., did not influence the design and functionality of the device when it was made. That's the claim of postulate 1, physics is the same in any inertial frame. If a manufacturer subcontracts components to another country, as long as they employ the same quality and measurement standards as the manufacturer, they can be confident the parts will fit when assembled.

As for 'false frames' and 'contradictions', the graphic is a slice through the light cone from an event E. It illustrates the statement, 'an event occurs once but can be perceived many times'. Since perception is always historical, or after the fact, the various perceptions occur at different times, none of which are simultaneous with E.
Distant events involve objects, but perception involves images of objects.

A simple example; viewing the details of the moon's surface. You are not 'really' that close, but the telescope modifies the 'image of the moon', AS IF you were closer.
The moon, telescope, viewer, and mental images, are all REAL.
 [ Invalid Attachment ]
Title: Re: Can we measure the one way speed of light?
Post by: David Cooper on 28/06/2018 19:44:54
Reciprocity requires that swapping A and B, yields the same results except B is moving in the opposite direction. The fact still remains that neither measure the speed of any object moving at the closing speeds.

The fact is that they always measure the speed of light relative to the other object as >c or <c if they consider themselves to be stationary. To measure it as c instead, they have to consider themselves to be moving, at which point the speed of light relative to them is >c and <c (in opposite directions, all the action taking place along a straight line). Your approach is simply to deny those measurements or deny their right to make them, and in doing so you make a mockery of mathematics.
Title: Re: Can we measure the one way speed of light?
Post by: David Cooper on 28/06/2018 20:17:22
If a rotating device made on Earth is working as designed, and put into a space probe and sent on a mission, the device continues to work as designed, despite moving at high speed relative to Earth. The motion of the device relative to any 'ether' is irrelevant, since measurements by and on the device are done within the device frame of reference.

The movement through the fabric of space is crucial - it's this that slows down the functionality to generate the measurements we get. Without that slowing of functionality, there would be no relativity, but that would also be impossible because it would result in light overtaking light on the same path.

Quote
The motion of the Earth, sun, galaxy, etc., did not influence the design and functionality of the device when it was made. That's the claim of postulate 1, physics is the same in any inertial frame. If a manufacturer subcontracts components to another country, as long as they employ the same quality and measurement standards as the manufacturer, they can be confident the parts will fit when assembled.

Welcome to relativity. No matter how fast you move the apparatus, the movement through space will automatically adjust the speed of functionality and will hide that movement through space.

Quote
As for 'false frames' and 'contradictions',

Which part of mathematics are you rejecting here? We know that at least one of the two objects Z and W has the property (while it maintains its current path and speed) that light passes it relative to it at >c in one direction and <c in the other. If a frame represents W as not having this property, then Z must have it, while if a frame represents Z as not having this property, then W must have it. It is not possible for neither of these objects to have that property, therefore at least one of those frames is misrepresenting reality. There is a clear contradiction in the claims of the two frames and they cannot both be true representations of reality. Mathematics says you are wrong. Do physicists genuinely respect mathematics or do they merely pretend to do so while waving two fingers at it under the desk?

Quote
The moon, telescope, viewer, and mental images, are all REAL.

Misrepresentations are fully real as themselves, but they still misrepresent the reality of the thing they represent. I could draw a politician with a bottom for a face - that would be a misrepresentation of the politician (in most cases), but it would be a real misrepresentation. The point, which you appear to be incapable of grasping is that frames of reference contradict each other fundamentally because they assert different speeds of light relative to the same object - if one frame represents reality correctly, every other frame misrepresents it.
Title: Re: Can we measure the one way speed of light?
Post by: guest4091 on 29/06/2018 19:52:22
David D. #;

In the graphic, B is moving parallel to A at speed b. B extends a stick forward with a mirror M on the far end. B measures the stick length d, to be 1 unit long.
B sends a signal to reflect from M and return. The graphic is the A description of events.

In the A frame, the round trip time would be 2d/c=2t.
A observes the round trip time for B to be:

Atr = (d/γ)(1/c)[1/(1-b)+1/(1+b)] = 2γd/c = 2γt

Where γ (gamma) is the ratio of a unit of time for a clock at rest to that of a moving clock.   
The term (d/γ) is the contracted length of d resulting from acceleration to its current speed. With adjustment for time dilation (red), Bt = Atr/γ = 2t.
The B-time will be the same as the rest frame A, and independent of speed b.
This is labeled as 'proper' time in the SR community. Since there is no issue of impropriety from the poor choice of words, I favor Einstein's simple and logical term, 'local' time. (it's about location!)

In your scenario, just substitute Y for A and W or Z for B.
Hopefully you noticed the use of (1-b) and (1+b) in the calculations, i.e. closing speeds,  which are definitely used in SR. No one argues that fact. Also notice these are calculations used by a remote observer A. They cannot be used by B, since he and all other inertial frames cannot determine their velocity through space.
If B assumes a pseudo rest frame, he would not expect the times out and back to be different.
 [ Invalid Attachment ]
Title: Re: Can we measure the one way speed of light?
Post by: guest4091 on 29/06/2018 19:55:34
David D. #;

In the graphic, B is moving parallel to A at speed b. B extends a stick forward with a mirror M on the far end. B measures the stick length d, to be 1 unit long.
B sends a signal to reflect from M and return. The graphic is the A description of events.

In the A frame, the round trip time would be 2d/c=2t.
A observes the round trip time for B to be:

Atr = (d/γ)(1/c)[1/(1-b)+1/(1+b)] = 2γd/c = 2γt

Where γ (gamma) is the ratio of a unit of time for a clock at rest to that of a moving clock.   
The term (d/γ) is the contracted length of d resulting from acceleration to its current speed. With adjustment for time dilation (red), Bt = Atr/γ = 2t.
The B-time will be the same as the rest frame A, and independent of speed b.
This is labeled as 'proper' time in the SR community. Since there is no issue of impropriety from the poor choice of words, I favor Einstein's simple and logical term, 'local' time. (it's about location!)

In your scenario, just substitute Y for A and W or Z for B.
Hopefully you noticed the use of (1-b) and (1+b) in the calculations, i.e. closing speeds,  which are definitely used in SR. No one argues that fact. Also notice these are calculations used by a remote observer A. They cannot be used by B, since he and all other inertial frames cannot determine their velocity through space.
If B assumes a pseudo rest frame, he would not expect the times out and back to be different.
 [ Invalid Attachment ]
Title: Re: Can we measure the one way speed of light?
Post by: yor_on on 29/06/2018 23:25:41
Phyti, the way you should use this is to hone your mind. If  you can simplify it, do it. But every question will make you think some more.
Title: Re: Can we measure the one way speed of light?
Post by: David Cooper on 29/06/2018 23:28:54
Phyti,

Your previous post doesn't address the issue in question, but simply avoids it. We aren't looking for a round trip time for X, but for the average speeds at which the two light pulses pass through the sectors of the cable relative to the material of those sectors while it's passing through them. You are supposed to be calculating values for these relative speeds (which I call R [for the red light] and B [for the blue]. Let's go through the argument again with the same thought experiment, but with a few added numbers. This may help determine where you part company with mathematics.

The course for the light to go round is a loop of flexible cable with an interior space and a mirrored lining which contains the light in that vacuum and ensures that it isn't slowed, though it may be easier just to visualise a loop of normal fibre-optic cable and to imagine that there are no delays caused by any kind of drag. This loop goes round over two pulleys at either end of a very long rod. The loop of cable is marked out into a hundred sectors named from S0 to S99. The pulleys are rotating such that all the material of the cable is moving round this very linear circuit at high speed. Sector S0 moves first to the right at 0.6c from one pulley to the other below the rod, then round the pulley, then it goes back to the left at 0.6c above the rod (if we assume the rod to be stationary). Note that the material of the cable must be length-contracted by this movement to 0.8 of its normal length, so we'd better have a telescopic rod which can contract to 0.8 of its normal length to match (so that we can set up the apparatus and then run it up to speed without the cable being deformed by stress. We also have object Z co-moving with the bottom part of the cable, and object W co-moving with the top part of the cable, so anything that we say about the speed of light relative to the bottom of the cable must also apply to object Z, and anything we say about the speed of light relative to the top of the cable must also apply to object W (and will continue to apply to them even if the rest of the apparatus is removed). Observer Y sits beside the left-hand pulley throughout, while observer X travels round the circuit with sector S0.

X leaves Y at the same time as pulses of light are emitted from S0 in opposite directions, red light going along the top (just over the rod) and blue going along the bottom parts of the cable (just under the rod), this happening once they've got clear of the pulley - initially they're moving upwards and downwards, but they're almost immediately sent round the corner such that both travel to the right. The light pulses eventually reach the far pulley simultaneously and continue their journey round the circuit, returning to the left until they reach Y simultaneously. They then set off to do more laps and repeat the exact same pattern each time.

What does observer X do in comparison to the light pulses? Well, he moves at 0.6c along the bottom of the circuit beside S0 (which he travels with throughout) and encounters a pulse of red light before reaching the right-hand pulley. How many circuits does X have to make before he is next back at Y at the same time as the light pulses return to Y? 5 x 0.6 = 3 (a whole number), so if light goes from one pulley to the other and back five times, X will have made the same trip three times. X has therefore encountered the red light pulse eight times, but has only encountered the blue light pulse twice. This means that the red light has passed through every sector of the loop four times more than the blue light has passed through them.

The speed of the red light relative to the material of the sectors it's passing through (while it's passing through them) can be calculated by measuring the length of the cable and timing how long it took for that light to pass through all the sectors. The length of the cable is either twice the distance between the pulleys, or else it is arguably longer than that because it's been length-contracted - you could allow X to measure it by wandering round the entire length of it and stopping to measure the length of each sector with a co-moving ruler. That longer length certainly doesn't suit you, so you'll want to use the shorter length, but I don't need the longer length either as the real length is the contracted length, and that's more than long enough to illustrate the point. If we call the distance between the pulleys 0.5L, then the length of the cable is L. If the time taken for light to travel from Y back to Y via the far pulley is t, then the speed of travel of the light is unsurprisingly 1/1=1=c. The time taken for X to travel from Y to the far pulley and back to Y is 5t, and during that time the red light has passed through all the sectors of the cable eight times, while the blue light has only passed through the sectors twice. The red light has therefore travelled through 8L of sectors in 5t, which gives us a speed relative to those sectors (while passing through them) of 1.6c, while the blue light has travelled 2L in 5t, which gives us an average relative speed of 0.4c. (What an amazing coincidence: that's the exact same values as we get just by adding or subtracting the speed of light to/from the speed of a moving object!)

You could assert that the length of the cable is four times shorter if you go round it anticlockwise than if you go clockwise, but there is nothing in SR that allows you to do so. It's really much better just to accept the measurements and stop trying to prop up a theory which conflicts with the facts.

Once we have established that the speed of light relative to some sectors is at least 1.6c , we know that either object W or object Z has the property that light passes it in one direction at a minimum speed of 1.6c relative to it. If a frame represents W as not having this property, then Z must have it, while if a frame represents Z as not having this property, then W must have it, so at least one of those two frames is a misrepresentation of reality because it asserts a falsehood about the speed of light relative to W or Z. When you extend this principle, it becomes clear that it's only possible for one frame be a true representation of reality, while all other frames misrepresent it, and that means we have an absolute frame.

What we see from SR adherents is a denial of the validity of all relative speeds other than c whenever light is one of the two things being compared. Z and W are moving relative to each other at 0.6c, but you are required by an authority to assert that light is moving relative to both of them at c in all directions, and in doing so, you necessarily generate the conclusion that Z and W must be co-moving, at which point you have to sweep that conclusion under the carpet and hope no one noticed. To tolerate these contradictions is a rejection of mathematics and I don't understand why anyone wants to defend a theory which generates such contradictions (particularly when there's another theory in existence which has been recognised as accounting for all the same experiments as SR and GR without generating any contradictions).

[When you press the top experts though, they attempt to escape into Minkowski Spacetime, at which point the speed of light becomes either zero or is infinite, depending on how you want to think about it - all the paths it follows are reduced to zero length. This leads to other fatal problems though, which we can explore later if anyone wants to take things in that direction - that's where the event-meshing failures come into play if you don't have a block universe, and it becomes impossible to generate a block universe under the rules of SR.]
Title: Re: Can we measure the one way speed of light?
Post by: David Cooper on 29/06/2018 23:39:34
Phyti, the way you should use this is to hone your mind. If  you can simplify it, do it. But every question will make you think some more.

There's a more important purpose than that, and that is to address the issue. He (and anyone else who holds the same position) should provide his own values for R and B and explain how he imagines they can be equal (or else admit that they are not equal). It shouldn't take pages of avoidance bloat to get round to that - it should have come in his next reply after I set out the original version of the experiment on page 2 in reply #77.
Title: Re: Can we measure the one way speed of light?
Post by: guest4091 on 30/06/2018 16:04:14
Phyti, the way you should use this is to hone your mind. If  you can simplify it, do it. But every question will make you think some more.

There's a more important purpose than that, and that is to address the issue. He (and anyone else who holds the same position) should provide his own values for R and B and explain how he imagines they can be equal (or else admit that they are not equal). It shouldn't take pages of avoidance bloat to get round to that - it should have come in his next reply after I set out the original version of the experiment on page 2 in reply #77.
There seems to be a history of David Cooper trying to discredit Relativity here.
No amount of evidence will convince you, you don't understand SR. The imagined 'problems' made me wonder, why your site is named 'magic school'.
This is tiresome and boring, so I have projects to work on, which are more constructive.
 
Title: Re: Can we measure the one way speed of light?
Post by: Le Repteux on 30/06/2018 16:43:34
Why the term discredit? Do you consider relativity to be so perfect that it can never be improved? It's not how science is expected to work. History of science shows that, like any other theory, relativity should one day become obsolete and be replaced by a more accurate idea. It is thus not relativity that is discredited here, but those who defend their belief with unfunded accusations instead of using logical ideas.
Title: Re: Can we measure the one way speed of light?
Post by: David Cooper on 30/06/2018 22:14:23
[There seems to be a history of David Cooper trying to discredit Relativity here.
No amount of evidence will convince you, you don't understand SR.

Sure - you must understand it better than me because you can't answer the questions I've put to you (and to others) even though I've provided all the answers to help you out. What I've just shown you is that the red light passes four times as much length of material in the same time as the blue light, and that means that the red light's speed relative to that material is four times that of the blue light relative to that material. You have no answer to that whatsoever, and SR has no magic formula to account for it either. For the average speeds of the red and blue light both to be c relative to the sectors they're passing through, you'd need some way to contract the length of the loop in one direction to half the measured length while doubling that same length in the other direction (negative length-contraction!), but the only known length contraction factor that can be applied here is the 0.8, and that's already been applied to the apparatus. So, all you can do is ignore the problem and pretend it isn't there. But at least you had a bit of a go at it and did eventually answer one question very reluctantly, so I congratulate you on that - you've done better than all the others on the SR side put together.

Quote
The imagined 'problems'...

How have you allowed yourself be blinded to the realities that I've placed before you? Do you deny that the light passes the sectors that it passes at a relative speed at all? What is that speed? This is really basic stuff, and it's absolutely shocking that no one from the SR camp (here and elsewhere ) is capable of taking it on. If it's c in both cases, then your numbers tell you that 1=4 (and that you're in conflict with mathematics). It's just applying bog-standard mathematics to the facts of bog-ordinary experiments and the production of clear conclusions driven by that maths. This shows though why so many leading mathematicians refuse point blank to comment on physics - they know that their maths is being misused in the most appalling ways, but they get a lot of grief if they dare to say so.

Quote
...made me wonder, why your site is named 'magic school'.

It's "Magic Schoolbook, and the explanation as to why is provided there. I hated school because it was just a children's prison where practically nothing was being taught. What I wanted was a special book that would teach me everything I was supposed to be learning at school so that I could get the hell on with it and never have to go back to that damned place ever again. I decided at five years old that I would write the book myself some day if no one else did it first - a book to liberate children from that idiotic school system that fails to teach and merely squanders childhood for nothing. But idiocy is hard to topple from power, as is clear everywhere you look - that's why the world's always been a hopeless mess in the way it's run. The only cure for that will come with AGI.

Quote
This is tiresome and boring, so I have projects to work on, which are more constructive.

I've shown you where SR fails, and your response is that you're bored. The truth is much simpler - you have no idea how to handle the argument because the argument is right. Just ask yourself why no one has been able to show that my numbers are wrong.
Title: Re: Can we measure the one way speed of light?
Post by: David Cooper on 01/07/2018 23:45:40
I don't normally respond to the same post twice, but this one's a special case.

There seems to be a history of David Cooper trying to discredit Relativity here.

There is, and the reason that I keep doing it is that certain people keep ignoring the faults with SR when I draw attention to them, never acknowledging them. "What contradictions?" they always say, even though the contradictions are manifest. The specific argument I've been using in this thread is aimed at making those contradictions even more obvious to those who have been systematically trained not to see them. This argument isn't even found on my relativity page because I'd always thought the contradictions were so obvious that there was no need to spell things out - most people new to the subject see them straight away because they haven't been brainwashed into being blind to them.

Quote
This is tiresome and boring, so I have projects to work on, which are more constructive.

I have important work that I could be getting on with too if the discussion wasn't dragged out by avoidance tactics? It's always tiresome and boring when a politician repeatedly fails to answer the questions they've been asked where they answer other questions instead which shed no light on the issue. Politicians do that because they're scoundrels. Scientists don't have the same need to avoid answering questions because they are honest pursuers of truth. So how should scientists deal with questions that are put to them? They should answer them directly. Here's how it ought to go (but never does) - I will post an imagined conversation between me (DC) and a scientist called Elf Mice (EM):-

DC: Is R>B?

EM: So; let's see if we can make R=B. If the red light passes through sector S0 at c relative to that sector, and at c relative to S1, and relative to S2, and S3, and so on all the way round the loop, and if the blue light passes through every sector at c relative to it too, then the time the red light takes to go through all the sectors eight times must be the same as the time the blue light takes to go through all the sectors eight times... unless the sectors are four times shorter in length when the red light goes through them than when the blue light does. That means, as you say, a length of stuff would need to be different for light passing it in different directions.

DC: And do you think that's possible?

EM: Assuming that light always passes everything at c, then no - the two lengths must match.

DC: is there absolutely no way that length contraction could be involved?

EM: Well, I'm not sure that you've entirely ruled that out, so let's look at it more closely. You say that you've already applied a length contraction factor of 0.8 because the loop of cable is going round the circuit at 0.6c, so if we're using the frame of reference in which Y is at rest, then that is indeed the only way that length contraction can apply here. If we use a different frame of reference though, what happens? Suppose we use the frame in which object Z is at rest... If we do this, the material in the lower part of the loop won't be contracted because it's at rest too in that frame.

DC: And the material in the top part of the loop?

EM: That will need to be much more strongly contracted.

DC: And that must affect the spacing between the pulleys. The rod length.

EM: Yes, I was just thinking that. We need to see if this R>B business applies for this frame too. The pulleys and rod must be moving to the left at 0.6c.

DC: Indeed, and more of the cable must be on the top part of the apparatus at any point in time than on the bottom part because it will bunch up with that stronger length contraction applying to it. The rod must still be telescoped to its shortened length, so this length contraction applying to it will further shrink it to 0.8 of that length.

EM: Right, and the contraction applying to the top part of the cable is worked out using relativistic velocity addition where we add 0.6c to 0.6c and then divide by 1 + 0.6x0.6, which gives us 0.88235c as the speed of movement of the sectors there.

DC: And that means a length contraction factor of 0.47059.

EM: I get that value too. But how do we apply these numbers...?

DC: Five laps for the light, but 8 or 2 laps of the material of the cable depending on the direction it goes round it.

EM: Ah yes - I see where you're going with that! The red light spends half its time passing material that's moving at 0.88c, and the other half of its time passing stationary material.

DC: No, that isn't right - the rod and pulleys are moving to the left, so the red light spends more time moving along the bottom part than the top and the blue light does the opposite.

EM: This is all devilishly complicated!

DC: That's why most people can't be bothered investigating this to check the facts. But before we go any further with the numbers, let's think a bit about that difference. The blue light spends most of its time travelling to the left through the top part where it's travelling at c past length-contracted material that's moving in the same direction at 0.88c, and it only spends a short time moving to the right along the bottom part where it's moving at c past material that's at rest and not contracted.

EM: Ah ha! So that light's spending most of its time passing length-contracted material, which means that it could in a way be shortening the course!

DC: Yes, but that's the blue light, and we'd want to lengthen the course for that if we're to find a get-out-of-jail-free card for SR. The red light spends very little time moving through the contracted material and spends the rest moving through material that's at rest, so on average it's spending less of its time on a contracted part of the course than the blue light.

EM: Which means this approach is futile.

DC: As it should be.

EM: I think I'll calculate those numbers anyway though, just to make sure they don't hold any surprises, and then I'll contact some of my colleagues to see if they have any ideas. Got to go for now though - I've got a machine doing an experiment here that'll blow up if I don't attend to it and flick a few switches.

DC: That's okay - nice talking to you, Mice Elf.

EM: Oh b***** it! Too late! Now see what you've gone and done by distracting me! I've just made a ruddy black hole, and it's eating my kitchen!

That's the way I expect a subject like this to be discussed - an honest exploration of the issues which tries to break an argument by taking it on instead of avoiding it, but I can't find anyone capable of holding such a co-operative conversation. All I get is, "You don't understand relativity", and yet I clearly understand it better than they do, not least because I actually implement it in software and can thereby check directly whether models really work when they apply their own rules instead of smuggling in other rules that aren't part of the model and pretending that a simulation of SR isn't just LET in disguise.
Title: Re: Can we measure the one way speed of light?
Post by: Kris Kuitkowski on 09/07/2018 13:11:53
Hi David,
I can see the topic of one way speed of light is still alive. If you recall our conversation about it, you insisted that length contraction of my rod (even moving at a very slow speed relative to the lasers) would falsify the results. I did not agree  but since done some thinking to make experiment more robust.
Instead of long rod lets have a solid square (each side length of the rod)  with small holes ABCD drilled close to each corner.  Let A'B'C'D'  be laser points aligned exactly with ABCD when stationary.
Similarly to our previous experiment let the square ABCD glide past lasers A'B'C'D'.
Since AB is perpendicular to the direction in which the square moves, there should be no length contraction of the distance AB regardless of the speed of the square. If there is a contraction of the distance AC (or BD), it should be relatively easy to measure it by measuring time of flight of a photon from A to B and comparing it with the time of flight from A to C.
Best regards,
Kris
Title: Re: Can we measure the one way speed of light?
Post by: David Cooper on 09/07/2018 21:08:12
I can see the topic of one way speed of light is still alive...

That doesn't belong in this thread, so you should either revive your old thread for this or start a new one in the same forum. Send a link to it in a PM to me if I don't respond to it within 24 hours. You can also delete your post from this thread (and I'll delete this reply).